Given name(s)
Diane
Family name
Sliwka
Degrees
MD

Everything We Say and Do: Hospitalists are leaders in designing inpatient experience

Article Type
Changed
Fri, 09/14/2018 - 12:00

Editor’s note: “Everything We Say and Do” is an informational series developed by SHM’s Patient Experience Committee to provide readers with thoughtful and actionable communication tactics that have great potential to positively affect patients’ experience of care. This column highlights key takeaways from the SHM track of the upcoming 2017 Cleveland Clinic Patient Experience Empathy and Innovation Summit, May 22-24. Three hospitalist leaders describe their approach to leading the design of the inpatient experience.

What we say and do, and why

Like many forms of care improvement, we have found that health care providers and patients alike engage most proactively when they are directly involved in codesigning an approach or intervention for improving the experience of care. Here are some examples of how hospitalists can be effective leaders in cocreating the inpatient experience with patients and interdisciplinary colleagues.

Dr. Diane Sliwka of the University of California, San Francisco
Dr. Diane Sliwka
Dr. Diane Sliwka: Design principles and systems improvement. Inspiring and sustaining effective improvement in patient experience and the work experience of the care team warrants rethinking of how we design our leadership, goals, and engagement of the people doing the work. Deliberate application of several principles has transformed improvement from being “another thing we have to do” to “the effective and engaging way we do things.” Effective improvement design has included visibility walls, streamlined goals and targets, access to real-time data, dyad leadership, huddles, and executive leader rounding. Through these methods, we nurture a culture of support for – and problem solving by – the people doing the work.

Dr. Patrick Kneeland of the University of Colorado, Aurora
Dr. Patrick Kneeland
Dr. Patrick Kneeland: User-centered design retreats.
We have implemented experience cocreation through user-centered design workshops that bring together patient voices, nurses, physicians, case managers, social workers, and pharmacists from a specific inpatient unit. Over half- or full-day sessions, the interdisciplinary team follows a facilitated “design thinking” approach to brainstorm, prototype, and refine new ideas. The outputs are brought back to the unit for implementation and ongoing refinement. Not only do innovative ideas emerge for enhancing the experience of care for both patients and providers, but there is also a measurable impact on unit culture and interdisciplinary collaboration.

Dr. Robert Hoffman of the University of Wisconsin-Madison
Dr. Robert Hoffman
Dr. Rob Hoffman: Partnering with patient and family advisers.
Working in close partnership with patient and family advisers (PFAs), we redesigned and implemented interdisciplinary bedside rounding in a way that puts the patient and family at the center of the care team. A multidisciplinary group including physicians, APPs, case managers, pharmacists, and PFAs created daily “care team visits” that bring, at a minimum, the nurse, provider and case manager to the beside daily. Key concepts we learned from our PFAs include having the nurse initiate the visit, minimizing the number of participants, clear introductions every time and focusing explicitly on what is most important to the patient that day. Our PFAs also actively participated in our training sessions for nurses and providers. Their stories and feedback at these trainings motivated attendees and helped everyone understand “why” we bring our conversations to the bedside. We have seen significant improvements in provider and nurse satisfaction with collaboration and unit level decision making and trends toward improved patient satisfaction with communication and teamwork.
 

Dr. Sliwka is medical director of patient and provider experience at University of California, San Francisco, Health; Dr. Kneeland is medical director for patient and provider experience at University of Colorado, Aurora, Hospital; Dr. Hoffman is medical director for patient relations at University of Wisconsin-Madison, Health.

Publications
Sections

Editor’s note: “Everything We Say and Do” is an informational series developed by SHM’s Patient Experience Committee to provide readers with thoughtful and actionable communication tactics that have great potential to positively affect patients’ experience of care. This column highlights key takeaways from the SHM track of the upcoming 2017 Cleveland Clinic Patient Experience Empathy and Innovation Summit, May 22-24. Three hospitalist leaders describe their approach to leading the design of the inpatient experience.

What we say and do, and why

Like many forms of care improvement, we have found that health care providers and patients alike engage most proactively when they are directly involved in codesigning an approach or intervention for improving the experience of care. Here are some examples of how hospitalists can be effective leaders in cocreating the inpatient experience with patients and interdisciplinary colleagues.

Dr. Diane Sliwka of the University of California, San Francisco
Dr. Diane Sliwka
Dr. Diane Sliwka: Design principles and systems improvement. Inspiring and sustaining effective improvement in patient experience and the work experience of the care team warrants rethinking of how we design our leadership, goals, and engagement of the people doing the work. Deliberate application of several principles has transformed improvement from being “another thing we have to do” to “the effective and engaging way we do things.” Effective improvement design has included visibility walls, streamlined goals and targets, access to real-time data, dyad leadership, huddles, and executive leader rounding. Through these methods, we nurture a culture of support for – and problem solving by – the people doing the work.

Dr. Patrick Kneeland of the University of Colorado, Aurora
Dr. Patrick Kneeland
Dr. Patrick Kneeland: User-centered design retreats.
We have implemented experience cocreation through user-centered design workshops that bring together patient voices, nurses, physicians, case managers, social workers, and pharmacists from a specific inpatient unit. Over half- or full-day sessions, the interdisciplinary team follows a facilitated “design thinking” approach to brainstorm, prototype, and refine new ideas. The outputs are brought back to the unit for implementation and ongoing refinement. Not only do innovative ideas emerge for enhancing the experience of care for both patients and providers, but there is also a measurable impact on unit culture and interdisciplinary collaboration.

Dr. Robert Hoffman of the University of Wisconsin-Madison
Dr. Robert Hoffman
Dr. Rob Hoffman: Partnering with patient and family advisers.
Working in close partnership with patient and family advisers (PFAs), we redesigned and implemented interdisciplinary bedside rounding in a way that puts the patient and family at the center of the care team. A multidisciplinary group including physicians, APPs, case managers, pharmacists, and PFAs created daily “care team visits” that bring, at a minimum, the nurse, provider and case manager to the beside daily. Key concepts we learned from our PFAs include having the nurse initiate the visit, minimizing the number of participants, clear introductions every time and focusing explicitly on what is most important to the patient that day. Our PFAs also actively participated in our training sessions for nurses and providers. Their stories and feedback at these trainings motivated attendees and helped everyone understand “why” we bring our conversations to the bedside. We have seen significant improvements in provider and nurse satisfaction with collaboration and unit level decision making and trends toward improved patient satisfaction with communication and teamwork.
 

Dr. Sliwka is medical director of patient and provider experience at University of California, San Francisco, Health; Dr. Kneeland is medical director for patient and provider experience at University of Colorado, Aurora, Hospital; Dr. Hoffman is medical director for patient relations at University of Wisconsin-Madison, Health.

Editor’s note: “Everything We Say and Do” is an informational series developed by SHM’s Patient Experience Committee to provide readers with thoughtful and actionable communication tactics that have great potential to positively affect patients’ experience of care. This column highlights key takeaways from the SHM track of the upcoming 2017 Cleveland Clinic Patient Experience Empathy and Innovation Summit, May 22-24. Three hospitalist leaders describe their approach to leading the design of the inpatient experience.

What we say and do, and why

Like many forms of care improvement, we have found that health care providers and patients alike engage most proactively when they are directly involved in codesigning an approach or intervention for improving the experience of care. Here are some examples of how hospitalists can be effective leaders in cocreating the inpatient experience with patients and interdisciplinary colleagues.

Dr. Diane Sliwka of the University of California, San Francisco
Dr. Diane Sliwka
Dr. Diane Sliwka: Design principles and systems improvement. Inspiring and sustaining effective improvement in patient experience and the work experience of the care team warrants rethinking of how we design our leadership, goals, and engagement of the people doing the work. Deliberate application of several principles has transformed improvement from being “another thing we have to do” to “the effective and engaging way we do things.” Effective improvement design has included visibility walls, streamlined goals and targets, access to real-time data, dyad leadership, huddles, and executive leader rounding. Through these methods, we nurture a culture of support for – and problem solving by – the people doing the work.

Dr. Patrick Kneeland of the University of Colorado, Aurora
Dr. Patrick Kneeland
Dr. Patrick Kneeland: User-centered design retreats.
We have implemented experience cocreation through user-centered design workshops that bring together patient voices, nurses, physicians, case managers, social workers, and pharmacists from a specific inpatient unit. Over half- or full-day sessions, the interdisciplinary team follows a facilitated “design thinking” approach to brainstorm, prototype, and refine new ideas. The outputs are brought back to the unit for implementation and ongoing refinement. Not only do innovative ideas emerge for enhancing the experience of care for both patients and providers, but there is also a measurable impact on unit culture and interdisciplinary collaboration.

Dr. Robert Hoffman of the University of Wisconsin-Madison
Dr. Robert Hoffman
Dr. Rob Hoffman: Partnering with patient and family advisers.
Working in close partnership with patient and family advisers (PFAs), we redesigned and implemented interdisciplinary bedside rounding in a way that puts the patient and family at the center of the care team. A multidisciplinary group including physicians, APPs, case managers, pharmacists, and PFAs created daily “care team visits” that bring, at a minimum, the nurse, provider and case manager to the beside daily. Key concepts we learned from our PFAs include having the nurse initiate the visit, minimizing the number of participants, clear introductions every time and focusing explicitly on what is most important to the patient that day. Our PFAs also actively participated in our training sessions for nurses and providers. Their stories and feedback at these trainings motivated attendees and helped everyone understand “why” we bring our conversations to the bedside. We have seen significant improvements in provider and nurse satisfaction with collaboration and unit level decision making and trends toward improved patient satisfaction with communication and teamwork.
 

Dr. Sliwka is medical director of patient and provider experience at University of California, San Francisco, Health; Dr. Kneeland is medical director for patient and provider experience at University of Colorado, Aurora, Hospital; Dr. Hoffman is medical director for patient relations at University of Wisconsin-Madison, Health.

Publications
Publications
Article Type
Sections
Disallow All Ads
Content Gating
No Gating (article Unlocked/Free)
Alternative CME

'Everything We Say and Do': Soliciting Goals from Our Patients and Their Families

Article Type
Changed
Fri, 09/14/2018 - 12:05
Display Headline
'Everything We Say and Do': Soliciting Goals from Our Patients and Their Families

Editor’s note: “Everything We Say and Do” is an informational series developed by SHM’s Patient Experience Committee to provide readers with thoughtful and actionable communication tactics that have great potential to positively impact patients’ experience of care. Each article will focus on how the contributor applies one ormore of the “key communication” tactics in practice to maintain provider accountability for “Everything we say and do that affects our patients’ thoughts, feelings and well-being.”

Read more from the “Everything We Say and Do” series.

What I Say and Do

Solicit a patient’s agenda and use active listening.

Why I Do It

The literature shows that, in most cases, doctors interrupt within 18 to 23 seconds and that when we interrupt, patients often never get back to what they were saying, the course of their story changes, and our diagnostic accuracy decreases. I also do it because I learn things that I wouldn’t otherwise know, and my patients feel heard and treated with respect. Listening has a healing effect, and in medicine, it can be equally if not more therapeutic than the medicines and clinical care we provide. I find that it helps me to be a more effective doctor, one who is helping my patient in the way that is most meaningful and helpful to them. It is very easy to navigate an encounter from the physician point of view and to make assumptions about what people want and need from me, but in reality, what is most important to me is not always what is most important to them.

Allowing patients to tell me what is important shows them respect and also sets me up for success as I am more likely to know and meet their needs. Doing this up front saves time by preventing the “doorknob” questions on the way out.  The human connection that follows keeps me connected to my purpose as a doctor. People often worry that listening will take too much time, but we know from the literature that most patients will talk for no more than 90 seconds. It’s really a very short amount of time for a gold mine of information.

How I Do It

Before I jump into my agenda, I make sure to know what is on the patient’s and family’s mind. What is most important to them to address? Once we have agreed on what we will be discussing or doing with our time in a way that includes both what I and the patient/family find important, I start by asking the patient to tell me everything about the first item at hand. I do not interrupt by asking questions, making comments, or “fixing.” I approach them with authentic curiosity, encouraging more without directing what they say.

I start with, “I’m here to talk to you about _____, but first, can you tell me what you’d like to make sure we talk about today?” Or, “Tell me a list of things that you’d like to make sure we talk about today.”

I follow that with, “What else?” until there is nothing else. Once we have negotiated what we will discuss, I say, “Tell me all about _______.” I do not interrupt or think about my response while I am listening. My only response is to use nonverbal continuers (“Uh huh,” “mmmm”), reflections (“That sounds really hard”), verbal continuers (“Tell me more”), empathic statements (“I can see why you would feel that way”), and body language that shows I am with them (sitting at eye level, facing them, looking at them rather than at my phone, pager or a computer screen.)

 

 

All of this happens before I jump into any of my own focused or clarifying questions.

Dr. Sliwka is medical director of patient and provider experience, medical director of the Goldman Medical Service, and associate clinical professor of medicine in the Division of Hospital Medicine at the UCSF Medical Center in San Francisco.

Table 1.

Issue
The Hospitalist - 2016(03)
Publications
Sections

Editor’s note: “Everything We Say and Do” is an informational series developed by SHM’s Patient Experience Committee to provide readers with thoughtful and actionable communication tactics that have great potential to positively impact patients’ experience of care. Each article will focus on how the contributor applies one ormore of the “key communication” tactics in practice to maintain provider accountability for “Everything we say and do that affects our patients’ thoughts, feelings and well-being.”

Read more from the “Everything We Say and Do” series.

What I Say and Do

Solicit a patient’s agenda and use active listening.

Why I Do It

The literature shows that, in most cases, doctors interrupt within 18 to 23 seconds and that when we interrupt, patients often never get back to what they were saying, the course of their story changes, and our diagnostic accuracy decreases. I also do it because I learn things that I wouldn’t otherwise know, and my patients feel heard and treated with respect. Listening has a healing effect, and in medicine, it can be equally if not more therapeutic than the medicines and clinical care we provide. I find that it helps me to be a more effective doctor, one who is helping my patient in the way that is most meaningful and helpful to them. It is very easy to navigate an encounter from the physician point of view and to make assumptions about what people want and need from me, but in reality, what is most important to me is not always what is most important to them.

Allowing patients to tell me what is important shows them respect and also sets me up for success as I am more likely to know and meet their needs. Doing this up front saves time by preventing the “doorknob” questions on the way out.  The human connection that follows keeps me connected to my purpose as a doctor. People often worry that listening will take too much time, but we know from the literature that most patients will talk for no more than 90 seconds. It’s really a very short amount of time for a gold mine of information.

How I Do It

Before I jump into my agenda, I make sure to know what is on the patient’s and family’s mind. What is most important to them to address? Once we have agreed on what we will be discussing or doing with our time in a way that includes both what I and the patient/family find important, I start by asking the patient to tell me everything about the first item at hand. I do not interrupt by asking questions, making comments, or “fixing.” I approach them with authentic curiosity, encouraging more without directing what they say.

I start with, “I’m here to talk to you about _____, but first, can you tell me what you’d like to make sure we talk about today?” Or, “Tell me a list of things that you’d like to make sure we talk about today.”

I follow that with, “What else?” until there is nothing else. Once we have negotiated what we will discuss, I say, “Tell me all about _______.” I do not interrupt or think about my response while I am listening. My only response is to use nonverbal continuers (“Uh huh,” “mmmm”), reflections (“That sounds really hard”), verbal continuers (“Tell me more”), empathic statements (“I can see why you would feel that way”), and body language that shows I am with them (sitting at eye level, facing them, looking at them rather than at my phone, pager or a computer screen.)

 

 

All of this happens before I jump into any of my own focused or clarifying questions.

Dr. Sliwka is medical director of patient and provider experience, medical director of the Goldman Medical Service, and associate clinical professor of medicine in the Division of Hospital Medicine at the UCSF Medical Center in San Francisco.

Table 1.

Editor’s note: “Everything We Say and Do” is an informational series developed by SHM’s Patient Experience Committee to provide readers with thoughtful and actionable communication tactics that have great potential to positively impact patients’ experience of care. Each article will focus on how the contributor applies one ormore of the “key communication” tactics in practice to maintain provider accountability for “Everything we say and do that affects our patients’ thoughts, feelings and well-being.”

Read more from the “Everything We Say and Do” series.

What I Say and Do

Solicit a patient’s agenda and use active listening.

Why I Do It

The literature shows that, in most cases, doctors interrupt within 18 to 23 seconds and that when we interrupt, patients often never get back to what they were saying, the course of their story changes, and our diagnostic accuracy decreases. I also do it because I learn things that I wouldn’t otherwise know, and my patients feel heard and treated with respect. Listening has a healing effect, and in medicine, it can be equally if not more therapeutic than the medicines and clinical care we provide. I find that it helps me to be a more effective doctor, one who is helping my patient in the way that is most meaningful and helpful to them. It is very easy to navigate an encounter from the physician point of view and to make assumptions about what people want and need from me, but in reality, what is most important to me is not always what is most important to them.

Allowing patients to tell me what is important shows them respect and also sets me up for success as I am more likely to know and meet their needs. Doing this up front saves time by preventing the “doorknob” questions on the way out.  The human connection that follows keeps me connected to my purpose as a doctor. People often worry that listening will take too much time, but we know from the literature that most patients will talk for no more than 90 seconds. It’s really a very short amount of time for a gold mine of information.

How I Do It

Before I jump into my agenda, I make sure to know what is on the patient’s and family’s mind. What is most important to them to address? Once we have agreed on what we will be discussing or doing with our time in a way that includes both what I and the patient/family find important, I start by asking the patient to tell me everything about the first item at hand. I do not interrupt by asking questions, making comments, or “fixing.” I approach them with authentic curiosity, encouraging more without directing what they say.

I start with, “I’m here to talk to you about _____, but first, can you tell me what you’d like to make sure we talk about today?” Or, “Tell me a list of things that you’d like to make sure we talk about today.”

I follow that with, “What else?” until there is nothing else. Once we have negotiated what we will discuss, I say, “Tell me all about _______.” I do not interrupt or think about my response while I am listening. My only response is to use nonverbal continuers (“Uh huh,” “mmmm”), reflections (“That sounds really hard”), verbal continuers (“Tell me more”), empathic statements (“I can see why you would feel that way”), and body language that shows I am with them (sitting at eye level, facing them, looking at them rather than at my phone, pager or a computer screen.)

 

 

All of this happens before I jump into any of my own focused or clarifying questions.

Dr. Sliwka is medical director of patient and provider experience, medical director of the Goldman Medical Service, and associate clinical professor of medicine in the Division of Hospital Medicine at the UCSF Medical Center in San Francisco.

Table 1.

Issue
The Hospitalist - 2016(03)
Issue
The Hospitalist - 2016(03)
Publications
Publications
Article Type
Display Headline
'Everything We Say and Do': Soliciting Goals from Our Patients and Their Families
Display Headline
'Everything We Say and Do': Soliciting Goals from Our Patients and Their Families
Sections
Disallow All Ads
Content Gating
No Gating (article Unlocked/Free)

Patient Satisfaction With Procedural Care

Article Type
Changed
Thu, 05/25/2017 - 21:15
Display Headline
Patient satisfaction with a hospitalist procedure service: Is bedside procedure teaching reassuring to patients?

In order to improve resident supervision and timeliness of invasive bedside procedures such as paracentesis, thoracentesis, and lumbar puncture, some academic medical centers have implemented procedure services that focus on providing high‐quality procedural care.1, 2

Procedure services have the potential to affect patient satisfaction, a key indicator in quality of care measurment.3 Having senior physicians present increases patient comfort during outpatient case presentations4 and improves patient satisfaction with explanations of tests and medications.5 However, we had concerns that teaching during a procedure may heighten patient anxiety. Patients are reluctant to be the first patient of a resident or medical student for a procedure,68 and patients are more likely to refuse consent to have a resident perform complex procedures.8 In previous studies, patient satisfaction with gynecological exams and flexible sigmoidoscopy performed by residents was comparable to satisfaction with those performed by staff physicians,9, 10 though in the case of flexible sigmoidoscopy, procedure duration was slightly longer.10 Few, if any, data describe bedside teaching or patient impressions of physician communication during procedures.

We carried out a prospective study of patient perceptions of the University of California San Francisco (UCSF) Hospitalist Procedure Service (HPS). Our study had the primary goal of understanding how our modelwhich involves bedside procedural teaching and feedback in real time (eg, as the procedure is performed)is perceived by patients.

Patients and Methods

Site

Our survey was carried out at UCSF Moffitt‐Long Hospital, a 560‐bed university teaching hospital and the primary university hospital for the University of California San Francisco. This study was reviewed and approved by the Committee on Human Research at UCSF.

Procedure Service

The HPS is composed of two interns who rotate for 2 weeks on a mandatory rotation performing the majority of the procedures done by the service. Every procedure is supervised by an attending hospitalist who has received extended training from interventional radiologists and emergency department ultrasound faculty. Patients are referred to the service by their primary admitting team. Interns receive procedure‐specific didactics, demonstration, and practice with procedure kits, supplemental readings, computer‐based procedure modules, and evidence‐based summaries of procedure‐related considerations. All interns also attend a half‐day procedure simulation session to review procedural and ultrasound techniques.

While interns obtain informed consent and prepare the patient for the procedure, the attending and intern team communicate the following points with each patient: 1) identification as the dedicated procedure team, separate from the primary team caring for the patient; 2) attending self‐identification as the supervisor; 3) attention to stepwise communication with the patient during the procedure; 4) attention to patient comfort throughout the procedure; 5) emphasis on patient safety through the use of time‐outs, sterile technique, and ultrasound when appropriate; and 6) the intention to discuss best practice and teach during the procedure.

All paracentesis and thoracentesis sites are marked by using bedside ultrasound (S‐Cath, SonoSite, Bothell, WA) guidance prior to and, if needed, during the procedure. Ultrasound is occasionally used for marking joint aspiration and lumbar puncture.11 Interns are responsible for making an initial site marking, which is then confirmed by the attending physician. Although not systematized, our service encourages the intern and attending to communicate about proper technique during the procedure itself. For example, attendings ask questions about technique based on evidence in the literature (eg, Why do you replace the stylet in a lumbar puncture needle prior to removal?) or about trouble shooting (eg, What would you do if the flow of ascites stops during this paracentesis?) and also correct any errors in technique (Recall the angle you intended to use based on the ultrasound view).

Patients

Patients are referred to the procedure service by their primary team; referrals are accepted for patients on all services at all levels of care, including the emergency department (ED) and the intensive care unit (ICU). Participants in this study were referred for one of our target procedures (paracentesis, thoracentesis, or lumbar puncture) between November 2008 and July 2009. Patients gave written consent for the supplemental survey independent of consent for the procedure. All consents and procedures were performed in a patient's hospital room and one family member was allowed to stay in the room if desired by the patient. After the completion of the procedure, the attending on the procedure service at the time, which included study authors D.S. and M.M., approached consecutive patients who spoke and read English and were deemed to have capacity to consent for their own procedure to be surveyed. Patients were considered to have capacity to consent based on commonly accepted criteria described in the literature.12, 13 Patients were also excluded if their procedure was performed by the attending alone, if they had repeated procedures done by the service, or if they were too altered or critically ill to participate in the survey.

Survey

Our survey was developed through identification of items reported in the literature,1421 as well as items newly developed for purposes of examining our primary aims. Newly developed questions focused on patients' satisfaction with major aspects of procedure performance as well as the quality and impact of communication with the patient and between members of the team. Two open‐text questions were included to allow patients to share what went well with the procedure as well as areas for improvement. The research team developed a pool of question items for potential inclusion in a patient satisfaction questionnaire. These items were then shown to a group of research‐oriented health professionals, who meet regularly to review academic research protocols. The group provided their opinions about the content and comprehension of the questions, and the written survey employed was a result of their revisions (see Appendix in Supporting Information online).

Written surveys were distributed to patients by the hospitalist attending on service following the procedure as permitted by patients' severity of illness and availability. Surveys were anonymous and self‐administered by the patient or a family member who was in the room for the procedure; all questions were voluntary. A nurse was made responsible for collecting the survey when possible. Survey results were entered into a database without identifiers, with limited demographic information; patient gender, age, and procedure type were included by the attending hospitalist at the end of the survey. A separate and more detailed procedure database was kept of all procedures performed and was used to record patient consent or reason for not consenting as well as documented receipt of a completed survey. This non‐anonymous database contained detailed supplemental information including patient age, level of care, referring service, presence of bloody fluid at any point during the procedure, and physician‐reported immediate complications at the bedside in free text.

Analysis

Reported immediate complications were classified into major and minor based on reported definitions in the literature.2226 Similar to previous studies, major immediate complications were defined as those requiring further procedural intervention, medical therapy, or both.27 Major complications were defined as: bleeding requiring transfusion, pneumothorax requiring a chest tube, respiratory failure, bowel perforation, cerebral herniation or shock, cerebrospinal fluid (CSF) leak requiring intervention, and transfer to a higher level of care. For patients receiving a thoracentesis, chart review was performed to determine the presence of a follow‐up chest x‐ray, the presence of a pneumothorax, or clinical evidence for re‐expansion pulmonary edema. We analyzed differences between respondents and non‐respondents using Chi‐square tests for categorical variables (gender, level of care, referring service, procedure type, bloody fluid, and immediate reported complications) and independent t tests for continuous variables (age).

After review of the open‐ended fields, responses were classified into the following categories: pain control, physician skill, professionalism, communication, symptom relief, procedure duration, and miscellaneous comments. Responses regarding patient perceptions of physician communication were dichotomized into positive (1 = Strongly Agree, 2 = Agree) and negative (3 = Neutral, 4 = Disagree, and 5 = Strongly Disagree), and independent t tests were used to determine the contribution of factors, such as age, while Chi‐square tests were used for the contribution of gender and procedure type. All statistical tests were performed by using the SAS statistical application program (version 9.2).

Results

Respondent Characteristics

Of 324 procedures performed by the HPS during the study period, 95 (29%) were eligible for consent. Of the 229 patients not eligible for consent, 32 (10%) were excluded because the procedure was performed by the attending alone, 76 (23%) lacked English proficiency or literacy, 66 (20%) had altered mental status, 32 (10%) were intubated and/or had severe illness precluding consent, and 23 (7%) were repeat procedures on patients who had previously completed the survey. Only two patients specifically requested an attending to perform the procedure after an introduction to the service. Of the 95 patients eligible for consent, 89 were consented for the survey, and 65 (68%) completed the survey. Of the six eligible, non‐consented patients, all were leaving the floor immediately following the procedure, and time did not allow for consent and survey distribution. There were no differences between eligible responders and nonresponders in age, gender, procedure, requesting service, presence of bloody fluid, or physician‐reported immediate complications (Table 1).

Baseline and Procedure Characteristics by Responder and Nonresponder (N = 89)
DemographicsRespondera (n = 65)Nonresponder (n = 24)
  • Differences between responders and non‐responders were not statistically significant. Abbreviation: SD, standard deviation.

Age, y [mean (SD)]55.4 (15.7)50.4 (17.4)
Male gender, n (%) male41 (63.1)11 (45.8)
Procedure, n (%)  
Paracentesis31 (47.7)10 (41.7)
Thoracentesis17 (25.8)6 (25.0)
Lumbar puncture15 (22.7)7 (29.2)
Arthrocentesis2 (3.0)1 (4.2)
Patient location, n (%)  
Floor47 (72.3)19 (79.2)
Step down/telemetry17 (26.1)3 (12.5)
Intensive care unit1 (1.5)2 (8.3)
Service requesting, n (%)  
Medicine29 (44.6)10 (41.7)
Cardiology6 (9.1)3 (12.5)
Liver transplant20 (30.3)7 (29.2)
Bone marrow transplant7 (10.6)1 (4.2)
Surgery01 (4.2)
Neurosurgery1 (1.5)1 (4.2)
Other2 (3.0)1 (4.2)
Reported presence of bloody fluid at any point in the procedure, n (%)9 (13.6)4 (16.7)
Other reported immediate complications  
Equipment malfunction2 (3.0)1 (4.2)
Significant cough/pleuritic pain1 (1.5)1 (4.2)
Transient oxygen desaturation1 (1.5)0
Ascites leak00
Hematoma00
Persistent bleeding00
Transfer to a higher level of care00

Complications

As complications would likely play a role in procedure satisfaction, we describe immediate complications for the study population. Of the 324 procedures performed during the study period, no patient had predefined major immediate complications. Upon further chart review of the 96 patients that had a thoracentesis performed, all had a follow‐up chest x‐ray and none suffered an iatrogenic pneumothorax or re‐expansion pulmonary edema. Minor immediate complications for the 324 procedures were reported as follows: postprocedure pain in four patients (1.2%), cough in nine patients (2.8%), five equipment malfunctions (1.5%), four ascites leaks (1.2%), and one incisional bleed requiring a suture for hemostasis (0.3%). There was no significant difference in complications between those consented for the survey and the total study population.

Procedure Satisfaction

More than 90% of patients were satisfied or very satisfied with most aspects of the procedure, including the informed consent process, pain control, expertise, and courtesy of physicians (Table 2). The percentage of patients satisfied with the duration of procedure (88%) was lower than for other measures of satisfaction. Of the 38 patients receiving therapeutic procedures, 34 (89%) were satisfied or highly satisfied with the improvement in symptoms following the procedure.

Procedure Satisfaction Measures (N = 65)
 Very Satisfied and Satisfied No. (%)Neutral No. (%)Dissatisfied and Very Dissatisfied No. (%)N/A No. (%)
Your overall procedure experience65 (100)0 (0)0 (0)0 (0)
Explanation of the procedure, risks, and benefits before the procedure64 (99)1 (2)0 (0)0 (0)
Pain control during the procedure60 (92)5 (8)0 (0)0 (0)
Expertise/skill of the physicians performing your procedure62 (95)3 (5)0 (0)0 (0)
Courtesy and bedside manner of the physicians performing your procedure65 (100)0 (0)0 (0)0 (0)
The time it took to perform your procedure57 (88)6 (9)0 (0)2 (3)
Improvement in your symptoms following this procedure, if applicable34 (52)7 (11)0 (0)24 (37)

When asked what went well with the procedure, 59 (91%) respondents provided additional comments and feedback. Each response was classified as described in the Methods section. Of the free text responses, 8 of the 59 patients (14%) commented on the attention to pain control (eg, The caring and attention to my pain was most important to me), 5 (8%) on the skills of the operators (Great examination of the entire stomach region with the ultrasound to ensure the best position of the catheter), 6 (10%) on the courtesy and professionalism of the team (eg, Courteous, team‐feeling, addressed my concerns), 9 (15%) on their communication with the team (eg, The doctors made me feel very comfortable before the procedure by laying out the plan and explaining each part of the procedure), and 8 (14%) on relief of their symptoms (eg, There was an almost immediate and significant improvement in my breathing, bloating, and pain). Twenty‐three of the 59 comments (39%) were categorized as miscellaneous (eg, All went great. I fell asleep).

When asked areas for improvement, 55 (85%) patients responded. Thirty‐three patients (60%) reported that nothing could be improved or they instructed the team to just keep doing what you are doing, while 22 (40%) patients expressed a concern. Responses were categorized in a similar fashion to the positive responses. Five of the 22 negative comments (23%) reported that the procedure took too long (eg, Procedure could have been shorter. I got tired sitting up), 4 (18%) commented on pain control (eg, The poke for marking my skin hurt more than the anesthetic. I was surprised), 6 (27%) felt communication was a problem (eg, Discuss the steps with the patient audibly, no whispering, speak clearly), and 7 (32%) had miscellaneous concerns (eg, Try not to do this procedure right after another one).

Physician Communication

Sixty‐four patients (98%) reported that the physicians performing their procedure communicated with each other during the procedure (Table 3). Although one patient did not feel that the physicians communicated with each other, he or she still answered the follow‐up questions regarding perceptions of physician communication. We excluded this patient from our analysis as his or her answers may not be reliable. The majority of patients (84%) reported this communication as reassuring and felt it was a normal part of procedure performance (94%). Those that did not agree that physician communication was reassuring did not differ in average age (P = 0.307), gender (P = 0.511), or procedure type (P = 0.562).

Physician Communications Measures (N = 64)
 Strongly Agree and Agree No. (%)Neutral No. (%)Disagree, and Strongly Disagree No. (%)
I felt that the physicians talking to each other about my procedure was reassuring to me54 (84)10 (16)0 (0)
Physicians talking to each other while doing a procedure is a normal part of doing a procedure60 (94)4 (6)0 (0)

Of all positive and negative comments, five specifically addressed communication between physicians. Most (four) reflected satisfaction with bedside teaching (eg, They discussed the procedure in a professional manner and eased my mind at all times) and with having an expert in the room (eg, [The team] discussed things like needle placement, which was nice because there was a second opinion right there in the room). Patients also felt that it was good to experience the teaching, with one patient reporting that the best part of the procedure was watching doctors learn from each other. Patients did not express specific reservations about bedside teaching, resident technique, or fear of complications in free text.

Discussion

Even though novice interns performed procedures and simultaneous bedside teaching, patient satisfaction with a teaching procedure service was high, and reported complication rates were low. In addition, a majority of patients found discussions related to teaching activities reassuring and potentially important to their perception of care quality. Analogous studies examining patient satisfaction with endoscopic care found similar rates of patient satisfaction with endoscopists' bedside manner, technical skills, and pain control, but these studies included sedated patients.21 Our results are unique, as we evaluated awake patients with attention to perception of bedside teaching with novice interns.

Our findings offer an alternative strategy for bedside procedural teaching that employs transparency in the use of an expert and a trainee to introduce patients to bedside teaching by experts, which is not common at many academic medical centers.28 Patients may have been reassured by a clear explanation of the role of the service and the providers involved as well as an assurance of expertise and attention to patient comfort and safety. In addition to patient satisfaction, this model has the potential to impact both the safety of bedside procedures and housestaff education around procedure performance. For example, pneumothorax rates using our procedure service model are lower than those published (0% vs. 4% for ultrasound‐guided thoracentesis and 8.5% for thoracentesis by less experienced clinicians).29

Providers may be reluctant to teach at the bedside of awake patients for fear of heightening patient anxiety over trainee inexperience. In the 1960s similar fears were raised over the concern for patient anxiety with bedside rounding,30 but later studies revealed these concerns to be largely unfounded. Instead, bedside rounds have been shown to positively influence patients' feelings about their hospital experience and their relationships with their physicians compared with patients whose case presentations were made in a conference room.31, 32 Given the opportunity to comment on areas for improvement, patients in our study specifically elaborated regarding pain control, communication, and efficiency problems. Although 16% of patients did not find the communication of physicians reassuring, none of the negative comments reflected problems with bedside teaching, but rather concepts such as desiring a better explanation of steps throughout the procedure. Specifically, patients desire better communication for unanticipated pain.

There are several limitations to this study. Lack of patient satisfaction data from a control group of patients whose procedures were performed by attendings or housestaff alone limits our ability to draw conclusions about our satisfaction scores. The scarce applicable literature offers only imperfect comparison data. Because hospitalists were not blinded to the survey, attending behavior may have been subject to a Hawthorne effect.33 Consenting patients after the procedure could have provided hospitalists with an opportunity to exclude patients who appeared less satisfied with their procedure; however, attempts were made to prevent this behavior by requiring strict accounting of why a patient was not consented for the study. Use of alternative personnel for consent such as nurses was explored, but was found not to be feasible due to limited resources. These data are only applicable to English‐speaking patients who are literate and well enough to complete a survey. It is not clear whether the experience for other patients would reflect the same outcomes. It is plausible that non‐English‐speaking patients might have more concerns about incomprehensible conversations taking place during their procedure. Although the surveys were anonymous and patients were told that the proceduralists would not see individual responses, responses may have been biased out of patient concern that their response might affect their care. Hospitalists obtaining consent, however, were careful to stress anonymity and the distinction between the primary team and the procedure team.

Academic hospitals are struggling with providing quality procedural care while balancing housestaff education and experience.28 With hospitalists playing an increasingly prominent role in housestaff education and patient satisfaction initiatives, the supervision of housestaff by trained hospitalist faculty may help meet both aims in the performance of invasive bedside procedures, particularly at institutions where simulation training resources are limited. Although concern may exist for potential patient anxiety with bedside teaching, our data demonstrate high levels of patient satisfaction with a hospitalist procedure service despite novice procedure performers and an emphasis on teaching during the procedure.

Files
References
  1. Smith CC,Gordon CE,Feller‐Kopman D, et al.Creation of an innovative inpatient medical procedure service and a method to evaluate house staff competency.J Gen Intern Med.2004;19(5 Pt 2):510513.
  2. Lucas BP,Asbury JK,Wang Y, et al.Impact of a bedside procedure service on general medicine inpatients: A firm‐based trial.J Hosp Med.2007;2(3):143149.
  3. Hospital Care Quality Information from the Consumer Perspective (HCAHPS).Quality Assurance Guidelines.Baltimore, MD:Centers for Medicare 113(8):657662.
  4. Lehmann LS,Brancati FL,Chen MC,Roter D,Dobs AS.The effect of bedside case presentations on patients' perceptions of their medical care.N Engl J Med.1997;336(16):11501155.
  5. Santen SA,Hemphill RR,Spanier CM,Fletcher ND.‘Sorry, it's my first time!’ Will patients consent to medical students learning procedures?Med Educ.2005;39(4):365369.
  6. Williams CT,Fost N.Ethical considerations surrounding first time procedures: a study and analysis of patient attitudes toward spinal taps by students.Kennedy Inst Ethics J.1992;2(3):217231.
  7. Santen SA,Hemphill RR,McDonald MF,Jo CO.Patients' willingness to allow residents to learn to practice medical procedures.Acad Med.2004;79(2):144147.
  8. Sheets KJ,Caruthers BS,Schwenk TL.Patient satisfaction with gynecologic care provided by family practice resident physicians.Fam Pract Res J.1991;11(4):421428.
  9. Jackson JL,Osgard E,Fincher RK.Resident participation in flexible sigmoidoscopy does not affect patient satisfaction.Am J Gastroenterol.2000;95(6):15631566.
  10. Peterson MA,Abele J.Bedside ultrasound for difficult lumbar puncture.J Emerg Med.2005;28(2):197200.
  11. Grisso T,Applebaum P.Conducting the Assessment. In:Assessing Competence to Consent to Treatment: A Guide for Physicians and Other Health Professionals.First Edition ed.New York, NY:Oxford University Press;1998:8091.
  12. Critchfield JM,Williams MV.Care of Ill, Socially Complicated Patients. In:Medical Management of Vulnerable 2007:407418.
  13. Mueller PR,Biswal S,Halpern EF,Kaufman JA,Lee MJ.Interventional radiologic procedures: patient anxiety, perception of pain, understanding of procedure, and satisfaction with medication‐‐a prospective study.Radiology.2000;215(3):684688.
  14. Hendriks AA,Vrielink MR,Smets EM,van Es SQ,De Haes JC.Improving the assessment of (in)patients' satisfaction with hospital care.Med Care.2001;39(3):270283.
  15. Nguyen Thi PL,Briancon S,Empereur F,Guillemin F.Factors determining inpatient satisfaction with care.Soc Sci Med.2002;54(4):493504.
  16. Hendriks AA,Oort FJ,Vrielink MR,Smets EM.Reliability and validity of the Satisfaction with Hospital Care Questionnaire.Int J Qual Health Care.2002;14(6):471482.
  17. Perneger TV,Kossovsky MP,Cathieni F,di Florio V,Burnand B.A randomized trial of four patient satisfaction questionnaires.Med Care.2003;41(12):13431352.
  18. Gonzalez N,Quintana JM,Bilbao A, et al.Development and validation of an in‐patient satisfaction questionnaire.Int J Qual Health Care.2005;17(6):465472.
  19. Maurer MH,Beck A,Hamm B,Gebauer B.Central venous port catheters: evaluation of patients' satisfaction with implantation under local anesthesia.J Vasc Access.2009;10(1):2732.
  20. Ko HH,Zhang H,Telford JJ,Enns R.Factors influencing patient satisfaction when undergoing endoscopic procedures.Gastrointest Endosc.2009;69(4):88391, quiz 891.e1.
  21. Grogan DR,Irwin RS,Channick R, et al.Complications associated with thoracentesis. A prospective, randomized study comparing three different methods.Arch Intern Med.1990;150(4):873877.
  22. De Gottardi A,Thevenot T,Spahr L, et al.Risk of complications after abdominal paracentesis in cirrhotic patients: a prospective study.Clin Gastroenterol Hepatol.2009;7(8):906909.
  23. Grabau CM,Crago SF,Hoff LK, et al.Performance standards for therapeutic abdominal paracentesis.Hepatology.2004;40(2):484488.
  24. Sempere AP,Berenguer‐Ruiz L,Lezcano‐Rodas M,Mira‐Berenguer F,Waez M.Lumbar puncture: its indications, contraindications, complications and technique.Rev Neurol.2007;45(7):433436.
  25. Allen SH.How to perform a lumbar puncture with the patient in the seated position.Br J Hosp Med (Lond).2006;67(3):M467.
  26. Durning SJ,Cation LJ,Jackson JL.Are commonly used resident measurements associated with procedural skills in internal medicine residency training?J Gen Intern Med.2007;22(3):357361.
  27. Mourad M,Kohlwes J,Maselli J,MERN Group,Auerbach AD.Supervising the Supervisors‐Procedural Training and Supervision in Internal Medicine Residency.J Gen Intern Med.2010.
  28. Gordon CE,Feller‐Kopman D,Balk EM,Smetana GW.Pneumothorax following thoracentesis: a systematic review and meta‐analysis.Arch Intern Med.2010;170(4):332339.
  29. Franzblau AN,Kairys D,Kaufman MR.The emotional impact of ward rounds.J Mt Sinai Hosp NY.1956;23(6):782803.
  30. Simons RJ,Baily RG,Zelis R,Zwillich CW.The physiologic and psychological effects of the bedside presentation.N Engl J Med.1989;321(18):12731275.
  31. Lehmann LS,Brancati FL,Chen MC,Roter D,Dobs AS.The effect of bedside case presentations on patients' perceptions of their medical care.N Engl J Med.1997;336(16):11501155.
  32. Holden JD.Hawthorne effects and research into professional practice.J Eval Clin Pract.2001;7(1):6570.
Article PDF
Issue
Journal of Hospital Medicine - 6(4)
Publications
Page Number
219-224
Legacy Keywords
graduate medical training, patient satisfaction, procedure education, supervision
Sections
Files
Files
Article PDF
Article PDF

In order to improve resident supervision and timeliness of invasive bedside procedures such as paracentesis, thoracentesis, and lumbar puncture, some academic medical centers have implemented procedure services that focus on providing high‐quality procedural care.1, 2

Procedure services have the potential to affect patient satisfaction, a key indicator in quality of care measurment.3 Having senior physicians present increases patient comfort during outpatient case presentations4 and improves patient satisfaction with explanations of tests and medications.5 However, we had concerns that teaching during a procedure may heighten patient anxiety. Patients are reluctant to be the first patient of a resident or medical student for a procedure,68 and patients are more likely to refuse consent to have a resident perform complex procedures.8 In previous studies, patient satisfaction with gynecological exams and flexible sigmoidoscopy performed by residents was comparable to satisfaction with those performed by staff physicians,9, 10 though in the case of flexible sigmoidoscopy, procedure duration was slightly longer.10 Few, if any, data describe bedside teaching or patient impressions of physician communication during procedures.

We carried out a prospective study of patient perceptions of the University of California San Francisco (UCSF) Hospitalist Procedure Service (HPS). Our study had the primary goal of understanding how our modelwhich involves bedside procedural teaching and feedback in real time (eg, as the procedure is performed)is perceived by patients.

Patients and Methods

Site

Our survey was carried out at UCSF Moffitt‐Long Hospital, a 560‐bed university teaching hospital and the primary university hospital for the University of California San Francisco. This study was reviewed and approved by the Committee on Human Research at UCSF.

Procedure Service

The HPS is composed of two interns who rotate for 2 weeks on a mandatory rotation performing the majority of the procedures done by the service. Every procedure is supervised by an attending hospitalist who has received extended training from interventional radiologists and emergency department ultrasound faculty. Patients are referred to the service by their primary admitting team. Interns receive procedure‐specific didactics, demonstration, and practice with procedure kits, supplemental readings, computer‐based procedure modules, and evidence‐based summaries of procedure‐related considerations. All interns also attend a half‐day procedure simulation session to review procedural and ultrasound techniques.

While interns obtain informed consent and prepare the patient for the procedure, the attending and intern team communicate the following points with each patient: 1) identification as the dedicated procedure team, separate from the primary team caring for the patient; 2) attending self‐identification as the supervisor; 3) attention to stepwise communication with the patient during the procedure; 4) attention to patient comfort throughout the procedure; 5) emphasis on patient safety through the use of time‐outs, sterile technique, and ultrasound when appropriate; and 6) the intention to discuss best practice and teach during the procedure.

All paracentesis and thoracentesis sites are marked by using bedside ultrasound (S‐Cath, SonoSite, Bothell, WA) guidance prior to and, if needed, during the procedure. Ultrasound is occasionally used for marking joint aspiration and lumbar puncture.11 Interns are responsible for making an initial site marking, which is then confirmed by the attending physician. Although not systematized, our service encourages the intern and attending to communicate about proper technique during the procedure itself. For example, attendings ask questions about technique based on evidence in the literature (eg, Why do you replace the stylet in a lumbar puncture needle prior to removal?) or about trouble shooting (eg, What would you do if the flow of ascites stops during this paracentesis?) and also correct any errors in technique (Recall the angle you intended to use based on the ultrasound view).

Patients

Patients are referred to the procedure service by their primary team; referrals are accepted for patients on all services at all levels of care, including the emergency department (ED) and the intensive care unit (ICU). Participants in this study were referred for one of our target procedures (paracentesis, thoracentesis, or lumbar puncture) between November 2008 and July 2009. Patients gave written consent for the supplemental survey independent of consent for the procedure. All consents and procedures were performed in a patient's hospital room and one family member was allowed to stay in the room if desired by the patient. After the completion of the procedure, the attending on the procedure service at the time, which included study authors D.S. and M.M., approached consecutive patients who spoke and read English and were deemed to have capacity to consent for their own procedure to be surveyed. Patients were considered to have capacity to consent based on commonly accepted criteria described in the literature.12, 13 Patients were also excluded if their procedure was performed by the attending alone, if they had repeated procedures done by the service, or if they were too altered or critically ill to participate in the survey.

Survey

Our survey was developed through identification of items reported in the literature,1421 as well as items newly developed for purposes of examining our primary aims. Newly developed questions focused on patients' satisfaction with major aspects of procedure performance as well as the quality and impact of communication with the patient and between members of the team. Two open‐text questions were included to allow patients to share what went well with the procedure as well as areas for improvement. The research team developed a pool of question items for potential inclusion in a patient satisfaction questionnaire. These items were then shown to a group of research‐oriented health professionals, who meet regularly to review academic research protocols. The group provided their opinions about the content and comprehension of the questions, and the written survey employed was a result of their revisions (see Appendix in Supporting Information online).

Written surveys were distributed to patients by the hospitalist attending on service following the procedure as permitted by patients' severity of illness and availability. Surveys were anonymous and self‐administered by the patient or a family member who was in the room for the procedure; all questions were voluntary. A nurse was made responsible for collecting the survey when possible. Survey results were entered into a database without identifiers, with limited demographic information; patient gender, age, and procedure type were included by the attending hospitalist at the end of the survey. A separate and more detailed procedure database was kept of all procedures performed and was used to record patient consent or reason for not consenting as well as documented receipt of a completed survey. This non‐anonymous database contained detailed supplemental information including patient age, level of care, referring service, presence of bloody fluid at any point during the procedure, and physician‐reported immediate complications at the bedside in free text.

Analysis

Reported immediate complications were classified into major and minor based on reported definitions in the literature.2226 Similar to previous studies, major immediate complications were defined as those requiring further procedural intervention, medical therapy, or both.27 Major complications were defined as: bleeding requiring transfusion, pneumothorax requiring a chest tube, respiratory failure, bowel perforation, cerebral herniation or shock, cerebrospinal fluid (CSF) leak requiring intervention, and transfer to a higher level of care. For patients receiving a thoracentesis, chart review was performed to determine the presence of a follow‐up chest x‐ray, the presence of a pneumothorax, or clinical evidence for re‐expansion pulmonary edema. We analyzed differences between respondents and non‐respondents using Chi‐square tests for categorical variables (gender, level of care, referring service, procedure type, bloody fluid, and immediate reported complications) and independent t tests for continuous variables (age).

After review of the open‐ended fields, responses were classified into the following categories: pain control, physician skill, professionalism, communication, symptom relief, procedure duration, and miscellaneous comments. Responses regarding patient perceptions of physician communication were dichotomized into positive (1 = Strongly Agree, 2 = Agree) and negative (3 = Neutral, 4 = Disagree, and 5 = Strongly Disagree), and independent t tests were used to determine the contribution of factors, such as age, while Chi‐square tests were used for the contribution of gender and procedure type. All statistical tests were performed by using the SAS statistical application program (version 9.2).

Results

Respondent Characteristics

Of 324 procedures performed by the HPS during the study period, 95 (29%) were eligible for consent. Of the 229 patients not eligible for consent, 32 (10%) were excluded because the procedure was performed by the attending alone, 76 (23%) lacked English proficiency or literacy, 66 (20%) had altered mental status, 32 (10%) were intubated and/or had severe illness precluding consent, and 23 (7%) were repeat procedures on patients who had previously completed the survey. Only two patients specifically requested an attending to perform the procedure after an introduction to the service. Of the 95 patients eligible for consent, 89 were consented for the survey, and 65 (68%) completed the survey. Of the six eligible, non‐consented patients, all were leaving the floor immediately following the procedure, and time did not allow for consent and survey distribution. There were no differences between eligible responders and nonresponders in age, gender, procedure, requesting service, presence of bloody fluid, or physician‐reported immediate complications (Table 1).

Baseline and Procedure Characteristics by Responder and Nonresponder (N = 89)
DemographicsRespondera (n = 65)Nonresponder (n = 24)
  • Differences between responders and non‐responders were not statistically significant. Abbreviation: SD, standard deviation.

Age, y [mean (SD)]55.4 (15.7)50.4 (17.4)
Male gender, n (%) male41 (63.1)11 (45.8)
Procedure, n (%)  
Paracentesis31 (47.7)10 (41.7)
Thoracentesis17 (25.8)6 (25.0)
Lumbar puncture15 (22.7)7 (29.2)
Arthrocentesis2 (3.0)1 (4.2)
Patient location, n (%)  
Floor47 (72.3)19 (79.2)
Step down/telemetry17 (26.1)3 (12.5)
Intensive care unit1 (1.5)2 (8.3)
Service requesting, n (%)  
Medicine29 (44.6)10 (41.7)
Cardiology6 (9.1)3 (12.5)
Liver transplant20 (30.3)7 (29.2)
Bone marrow transplant7 (10.6)1 (4.2)
Surgery01 (4.2)
Neurosurgery1 (1.5)1 (4.2)
Other2 (3.0)1 (4.2)
Reported presence of bloody fluid at any point in the procedure, n (%)9 (13.6)4 (16.7)
Other reported immediate complications  
Equipment malfunction2 (3.0)1 (4.2)
Significant cough/pleuritic pain1 (1.5)1 (4.2)
Transient oxygen desaturation1 (1.5)0
Ascites leak00
Hematoma00
Persistent bleeding00
Transfer to a higher level of care00

Complications

As complications would likely play a role in procedure satisfaction, we describe immediate complications for the study population. Of the 324 procedures performed during the study period, no patient had predefined major immediate complications. Upon further chart review of the 96 patients that had a thoracentesis performed, all had a follow‐up chest x‐ray and none suffered an iatrogenic pneumothorax or re‐expansion pulmonary edema. Minor immediate complications for the 324 procedures were reported as follows: postprocedure pain in four patients (1.2%), cough in nine patients (2.8%), five equipment malfunctions (1.5%), four ascites leaks (1.2%), and one incisional bleed requiring a suture for hemostasis (0.3%). There was no significant difference in complications between those consented for the survey and the total study population.

Procedure Satisfaction

More than 90% of patients were satisfied or very satisfied with most aspects of the procedure, including the informed consent process, pain control, expertise, and courtesy of physicians (Table 2). The percentage of patients satisfied with the duration of procedure (88%) was lower than for other measures of satisfaction. Of the 38 patients receiving therapeutic procedures, 34 (89%) were satisfied or highly satisfied with the improvement in symptoms following the procedure.

Procedure Satisfaction Measures (N = 65)
 Very Satisfied and Satisfied No. (%)Neutral No. (%)Dissatisfied and Very Dissatisfied No. (%)N/A No. (%)
Your overall procedure experience65 (100)0 (0)0 (0)0 (0)
Explanation of the procedure, risks, and benefits before the procedure64 (99)1 (2)0 (0)0 (0)
Pain control during the procedure60 (92)5 (8)0 (0)0 (0)
Expertise/skill of the physicians performing your procedure62 (95)3 (5)0 (0)0 (0)
Courtesy and bedside manner of the physicians performing your procedure65 (100)0 (0)0 (0)0 (0)
The time it took to perform your procedure57 (88)6 (9)0 (0)2 (3)
Improvement in your symptoms following this procedure, if applicable34 (52)7 (11)0 (0)24 (37)

When asked what went well with the procedure, 59 (91%) respondents provided additional comments and feedback. Each response was classified as described in the Methods section. Of the free text responses, 8 of the 59 patients (14%) commented on the attention to pain control (eg, The caring and attention to my pain was most important to me), 5 (8%) on the skills of the operators (Great examination of the entire stomach region with the ultrasound to ensure the best position of the catheter), 6 (10%) on the courtesy and professionalism of the team (eg, Courteous, team‐feeling, addressed my concerns), 9 (15%) on their communication with the team (eg, The doctors made me feel very comfortable before the procedure by laying out the plan and explaining each part of the procedure), and 8 (14%) on relief of their symptoms (eg, There was an almost immediate and significant improvement in my breathing, bloating, and pain). Twenty‐three of the 59 comments (39%) were categorized as miscellaneous (eg, All went great. I fell asleep).

When asked areas for improvement, 55 (85%) patients responded. Thirty‐three patients (60%) reported that nothing could be improved or they instructed the team to just keep doing what you are doing, while 22 (40%) patients expressed a concern. Responses were categorized in a similar fashion to the positive responses. Five of the 22 negative comments (23%) reported that the procedure took too long (eg, Procedure could have been shorter. I got tired sitting up), 4 (18%) commented on pain control (eg, The poke for marking my skin hurt more than the anesthetic. I was surprised), 6 (27%) felt communication was a problem (eg, Discuss the steps with the patient audibly, no whispering, speak clearly), and 7 (32%) had miscellaneous concerns (eg, Try not to do this procedure right after another one).

Physician Communication

Sixty‐four patients (98%) reported that the physicians performing their procedure communicated with each other during the procedure (Table 3). Although one patient did not feel that the physicians communicated with each other, he or she still answered the follow‐up questions regarding perceptions of physician communication. We excluded this patient from our analysis as his or her answers may not be reliable. The majority of patients (84%) reported this communication as reassuring and felt it was a normal part of procedure performance (94%). Those that did not agree that physician communication was reassuring did not differ in average age (P = 0.307), gender (P = 0.511), or procedure type (P = 0.562).

Physician Communications Measures (N = 64)
 Strongly Agree and Agree No. (%)Neutral No. (%)Disagree, and Strongly Disagree No. (%)
I felt that the physicians talking to each other about my procedure was reassuring to me54 (84)10 (16)0 (0)
Physicians talking to each other while doing a procedure is a normal part of doing a procedure60 (94)4 (6)0 (0)

Of all positive and negative comments, five specifically addressed communication between physicians. Most (four) reflected satisfaction with bedside teaching (eg, They discussed the procedure in a professional manner and eased my mind at all times) and with having an expert in the room (eg, [The team] discussed things like needle placement, which was nice because there was a second opinion right there in the room). Patients also felt that it was good to experience the teaching, with one patient reporting that the best part of the procedure was watching doctors learn from each other. Patients did not express specific reservations about bedside teaching, resident technique, or fear of complications in free text.

Discussion

Even though novice interns performed procedures and simultaneous bedside teaching, patient satisfaction with a teaching procedure service was high, and reported complication rates were low. In addition, a majority of patients found discussions related to teaching activities reassuring and potentially important to their perception of care quality. Analogous studies examining patient satisfaction with endoscopic care found similar rates of patient satisfaction with endoscopists' bedside manner, technical skills, and pain control, but these studies included sedated patients.21 Our results are unique, as we evaluated awake patients with attention to perception of bedside teaching with novice interns.

Our findings offer an alternative strategy for bedside procedural teaching that employs transparency in the use of an expert and a trainee to introduce patients to bedside teaching by experts, which is not common at many academic medical centers.28 Patients may have been reassured by a clear explanation of the role of the service and the providers involved as well as an assurance of expertise and attention to patient comfort and safety. In addition to patient satisfaction, this model has the potential to impact both the safety of bedside procedures and housestaff education around procedure performance. For example, pneumothorax rates using our procedure service model are lower than those published (0% vs. 4% for ultrasound‐guided thoracentesis and 8.5% for thoracentesis by less experienced clinicians).29

Providers may be reluctant to teach at the bedside of awake patients for fear of heightening patient anxiety over trainee inexperience. In the 1960s similar fears were raised over the concern for patient anxiety with bedside rounding,30 but later studies revealed these concerns to be largely unfounded. Instead, bedside rounds have been shown to positively influence patients' feelings about their hospital experience and their relationships with their physicians compared with patients whose case presentations were made in a conference room.31, 32 Given the opportunity to comment on areas for improvement, patients in our study specifically elaborated regarding pain control, communication, and efficiency problems. Although 16% of patients did not find the communication of physicians reassuring, none of the negative comments reflected problems with bedside teaching, but rather concepts such as desiring a better explanation of steps throughout the procedure. Specifically, patients desire better communication for unanticipated pain.

There are several limitations to this study. Lack of patient satisfaction data from a control group of patients whose procedures were performed by attendings or housestaff alone limits our ability to draw conclusions about our satisfaction scores. The scarce applicable literature offers only imperfect comparison data. Because hospitalists were not blinded to the survey, attending behavior may have been subject to a Hawthorne effect.33 Consenting patients after the procedure could have provided hospitalists with an opportunity to exclude patients who appeared less satisfied with their procedure; however, attempts were made to prevent this behavior by requiring strict accounting of why a patient was not consented for the study. Use of alternative personnel for consent such as nurses was explored, but was found not to be feasible due to limited resources. These data are only applicable to English‐speaking patients who are literate and well enough to complete a survey. It is not clear whether the experience for other patients would reflect the same outcomes. It is plausible that non‐English‐speaking patients might have more concerns about incomprehensible conversations taking place during their procedure. Although the surveys were anonymous and patients were told that the proceduralists would not see individual responses, responses may have been biased out of patient concern that their response might affect their care. Hospitalists obtaining consent, however, were careful to stress anonymity and the distinction between the primary team and the procedure team.

Academic hospitals are struggling with providing quality procedural care while balancing housestaff education and experience.28 With hospitalists playing an increasingly prominent role in housestaff education and patient satisfaction initiatives, the supervision of housestaff by trained hospitalist faculty may help meet both aims in the performance of invasive bedside procedures, particularly at institutions where simulation training resources are limited. Although concern may exist for potential patient anxiety with bedside teaching, our data demonstrate high levels of patient satisfaction with a hospitalist procedure service despite novice procedure performers and an emphasis on teaching during the procedure.

In order to improve resident supervision and timeliness of invasive bedside procedures such as paracentesis, thoracentesis, and lumbar puncture, some academic medical centers have implemented procedure services that focus on providing high‐quality procedural care.1, 2

Procedure services have the potential to affect patient satisfaction, a key indicator in quality of care measurment.3 Having senior physicians present increases patient comfort during outpatient case presentations4 and improves patient satisfaction with explanations of tests and medications.5 However, we had concerns that teaching during a procedure may heighten patient anxiety. Patients are reluctant to be the first patient of a resident or medical student for a procedure,68 and patients are more likely to refuse consent to have a resident perform complex procedures.8 In previous studies, patient satisfaction with gynecological exams and flexible sigmoidoscopy performed by residents was comparable to satisfaction with those performed by staff physicians,9, 10 though in the case of flexible sigmoidoscopy, procedure duration was slightly longer.10 Few, if any, data describe bedside teaching or patient impressions of physician communication during procedures.

We carried out a prospective study of patient perceptions of the University of California San Francisco (UCSF) Hospitalist Procedure Service (HPS). Our study had the primary goal of understanding how our modelwhich involves bedside procedural teaching and feedback in real time (eg, as the procedure is performed)is perceived by patients.

Patients and Methods

Site

Our survey was carried out at UCSF Moffitt‐Long Hospital, a 560‐bed university teaching hospital and the primary university hospital for the University of California San Francisco. This study was reviewed and approved by the Committee on Human Research at UCSF.

Procedure Service

The HPS is composed of two interns who rotate for 2 weeks on a mandatory rotation performing the majority of the procedures done by the service. Every procedure is supervised by an attending hospitalist who has received extended training from interventional radiologists and emergency department ultrasound faculty. Patients are referred to the service by their primary admitting team. Interns receive procedure‐specific didactics, demonstration, and practice with procedure kits, supplemental readings, computer‐based procedure modules, and evidence‐based summaries of procedure‐related considerations. All interns also attend a half‐day procedure simulation session to review procedural and ultrasound techniques.

While interns obtain informed consent and prepare the patient for the procedure, the attending and intern team communicate the following points with each patient: 1) identification as the dedicated procedure team, separate from the primary team caring for the patient; 2) attending self‐identification as the supervisor; 3) attention to stepwise communication with the patient during the procedure; 4) attention to patient comfort throughout the procedure; 5) emphasis on patient safety through the use of time‐outs, sterile technique, and ultrasound when appropriate; and 6) the intention to discuss best practice and teach during the procedure.

All paracentesis and thoracentesis sites are marked by using bedside ultrasound (S‐Cath, SonoSite, Bothell, WA) guidance prior to and, if needed, during the procedure. Ultrasound is occasionally used for marking joint aspiration and lumbar puncture.11 Interns are responsible for making an initial site marking, which is then confirmed by the attending physician. Although not systematized, our service encourages the intern and attending to communicate about proper technique during the procedure itself. For example, attendings ask questions about technique based on evidence in the literature (eg, Why do you replace the stylet in a lumbar puncture needle prior to removal?) or about trouble shooting (eg, What would you do if the flow of ascites stops during this paracentesis?) and also correct any errors in technique (Recall the angle you intended to use based on the ultrasound view).

Patients

Patients are referred to the procedure service by their primary team; referrals are accepted for patients on all services at all levels of care, including the emergency department (ED) and the intensive care unit (ICU). Participants in this study were referred for one of our target procedures (paracentesis, thoracentesis, or lumbar puncture) between November 2008 and July 2009. Patients gave written consent for the supplemental survey independent of consent for the procedure. All consents and procedures were performed in a patient's hospital room and one family member was allowed to stay in the room if desired by the patient. After the completion of the procedure, the attending on the procedure service at the time, which included study authors D.S. and M.M., approached consecutive patients who spoke and read English and were deemed to have capacity to consent for their own procedure to be surveyed. Patients were considered to have capacity to consent based on commonly accepted criteria described in the literature.12, 13 Patients were also excluded if their procedure was performed by the attending alone, if they had repeated procedures done by the service, or if they were too altered or critically ill to participate in the survey.

Survey

Our survey was developed through identification of items reported in the literature,1421 as well as items newly developed for purposes of examining our primary aims. Newly developed questions focused on patients' satisfaction with major aspects of procedure performance as well as the quality and impact of communication with the patient and between members of the team. Two open‐text questions were included to allow patients to share what went well with the procedure as well as areas for improvement. The research team developed a pool of question items for potential inclusion in a patient satisfaction questionnaire. These items were then shown to a group of research‐oriented health professionals, who meet regularly to review academic research protocols. The group provided their opinions about the content and comprehension of the questions, and the written survey employed was a result of their revisions (see Appendix in Supporting Information online).

Written surveys were distributed to patients by the hospitalist attending on service following the procedure as permitted by patients' severity of illness and availability. Surveys were anonymous and self‐administered by the patient or a family member who was in the room for the procedure; all questions were voluntary. A nurse was made responsible for collecting the survey when possible. Survey results were entered into a database without identifiers, with limited demographic information; patient gender, age, and procedure type were included by the attending hospitalist at the end of the survey. A separate and more detailed procedure database was kept of all procedures performed and was used to record patient consent or reason for not consenting as well as documented receipt of a completed survey. This non‐anonymous database contained detailed supplemental information including patient age, level of care, referring service, presence of bloody fluid at any point during the procedure, and physician‐reported immediate complications at the bedside in free text.

Analysis

Reported immediate complications were classified into major and minor based on reported definitions in the literature.2226 Similar to previous studies, major immediate complications were defined as those requiring further procedural intervention, medical therapy, or both.27 Major complications were defined as: bleeding requiring transfusion, pneumothorax requiring a chest tube, respiratory failure, bowel perforation, cerebral herniation or shock, cerebrospinal fluid (CSF) leak requiring intervention, and transfer to a higher level of care. For patients receiving a thoracentesis, chart review was performed to determine the presence of a follow‐up chest x‐ray, the presence of a pneumothorax, or clinical evidence for re‐expansion pulmonary edema. We analyzed differences between respondents and non‐respondents using Chi‐square tests for categorical variables (gender, level of care, referring service, procedure type, bloody fluid, and immediate reported complications) and independent t tests for continuous variables (age).

After review of the open‐ended fields, responses were classified into the following categories: pain control, physician skill, professionalism, communication, symptom relief, procedure duration, and miscellaneous comments. Responses regarding patient perceptions of physician communication were dichotomized into positive (1 = Strongly Agree, 2 = Agree) and negative (3 = Neutral, 4 = Disagree, and 5 = Strongly Disagree), and independent t tests were used to determine the contribution of factors, such as age, while Chi‐square tests were used for the contribution of gender and procedure type. All statistical tests were performed by using the SAS statistical application program (version 9.2).

Results

Respondent Characteristics

Of 324 procedures performed by the HPS during the study period, 95 (29%) were eligible for consent. Of the 229 patients not eligible for consent, 32 (10%) were excluded because the procedure was performed by the attending alone, 76 (23%) lacked English proficiency or literacy, 66 (20%) had altered mental status, 32 (10%) were intubated and/or had severe illness precluding consent, and 23 (7%) were repeat procedures on patients who had previously completed the survey. Only two patients specifically requested an attending to perform the procedure after an introduction to the service. Of the 95 patients eligible for consent, 89 were consented for the survey, and 65 (68%) completed the survey. Of the six eligible, non‐consented patients, all were leaving the floor immediately following the procedure, and time did not allow for consent and survey distribution. There were no differences between eligible responders and nonresponders in age, gender, procedure, requesting service, presence of bloody fluid, or physician‐reported immediate complications (Table 1).

Baseline and Procedure Characteristics by Responder and Nonresponder (N = 89)
DemographicsRespondera (n = 65)Nonresponder (n = 24)
  • Differences between responders and non‐responders were not statistically significant. Abbreviation: SD, standard deviation.

Age, y [mean (SD)]55.4 (15.7)50.4 (17.4)
Male gender, n (%) male41 (63.1)11 (45.8)
Procedure, n (%)  
Paracentesis31 (47.7)10 (41.7)
Thoracentesis17 (25.8)6 (25.0)
Lumbar puncture15 (22.7)7 (29.2)
Arthrocentesis2 (3.0)1 (4.2)
Patient location, n (%)  
Floor47 (72.3)19 (79.2)
Step down/telemetry17 (26.1)3 (12.5)
Intensive care unit1 (1.5)2 (8.3)
Service requesting, n (%)  
Medicine29 (44.6)10 (41.7)
Cardiology6 (9.1)3 (12.5)
Liver transplant20 (30.3)7 (29.2)
Bone marrow transplant7 (10.6)1 (4.2)
Surgery01 (4.2)
Neurosurgery1 (1.5)1 (4.2)
Other2 (3.0)1 (4.2)
Reported presence of bloody fluid at any point in the procedure, n (%)9 (13.6)4 (16.7)
Other reported immediate complications  
Equipment malfunction2 (3.0)1 (4.2)
Significant cough/pleuritic pain1 (1.5)1 (4.2)
Transient oxygen desaturation1 (1.5)0
Ascites leak00
Hematoma00
Persistent bleeding00
Transfer to a higher level of care00

Complications

As complications would likely play a role in procedure satisfaction, we describe immediate complications for the study population. Of the 324 procedures performed during the study period, no patient had predefined major immediate complications. Upon further chart review of the 96 patients that had a thoracentesis performed, all had a follow‐up chest x‐ray and none suffered an iatrogenic pneumothorax or re‐expansion pulmonary edema. Minor immediate complications for the 324 procedures were reported as follows: postprocedure pain in four patients (1.2%), cough in nine patients (2.8%), five equipment malfunctions (1.5%), four ascites leaks (1.2%), and one incisional bleed requiring a suture for hemostasis (0.3%). There was no significant difference in complications between those consented for the survey and the total study population.

Procedure Satisfaction

More than 90% of patients were satisfied or very satisfied with most aspects of the procedure, including the informed consent process, pain control, expertise, and courtesy of physicians (Table 2). The percentage of patients satisfied with the duration of procedure (88%) was lower than for other measures of satisfaction. Of the 38 patients receiving therapeutic procedures, 34 (89%) were satisfied or highly satisfied with the improvement in symptoms following the procedure.

Procedure Satisfaction Measures (N = 65)
 Very Satisfied and Satisfied No. (%)Neutral No. (%)Dissatisfied and Very Dissatisfied No. (%)N/A No. (%)
Your overall procedure experience65 (100)0 (0)0 (0)0 (0)
Explanation of the procedure, risks, and benefits before the procedure64 (99)1 (2)0 (0)0 (0)
Pain control during the procedure60 (92)5 (8)0 (0)0 (0)
Expertise/skill of the physicians performing your procedure62 (95)3 (5)0 (0)0 (0)
Courtesy and bedside manner of the physicians performing your procedure65 (100)0 (0)0 (0)0 (0)
The time it took to perform your procedure57 (88)6 (9)0 (0)2 (3)
Improvement in your symptoms following this procedure, if applicable34 (52)7 (11)0 (0)24 (37)

When asked what went well with the procedure, 59 (91%) respondents provided additional comments and feedback. Each response was classified as described in the Methods section. Of the free text responses, 8 of the 59 patients (14%) commented on the attention to pain control (eg, The caring and attention to my pain was most important to me), 5 (8%) on the skills of the operators (Great examination of the entire stomach region with the ultrasound to ensure the best position of the catheter), 6 (10%) on the courtesy and professionalism of the team (eg, Courteous, team‐feeling, addressed my concerns), 9 (15%) on their communication with the team (eg, The doctors made me feel very comfortable before the procedure by laying out the plan and explaining each part of the procedure), and 8 (14%) on relief of their symptoms (eg, There was an almost immediate and significant improvement in my breathing, bloating, and pain). Twenty‐three of the 59 comments (39%) were categorized as miscellaneous (eg, All went great. I fell asleep).

When asked areas for improvement, 55 (85%) patients responded. Thirty‐three patients (60%) reported that nothing could be improved or they instructed the team to just keep doing what you are doing, while 22 (40%) patients expressed a concern. Responses were categorized in a similar fashion to the positive responses. Five of the 22 negative comments (23%) reported that the procedure took too long (eg, Procedure could have been shorter. I got tired sitting up), 4 (18%) commented on pain control (eg, The poke for marking my skin hurt more than the anesthetic. I was surprised), 6 (27%) felt communication was a problem (eg, Discuss the steps with the patient audibly, no whispering, speak clearly), and 7 (32%) had miscellaneous concerns (eg, Try not to do this procedure right after another one).

Physician Communication

Sixty‐four patients (98%) reported that the physicians performing their procedure communicated with each other during the procedure (Table 3). Although one patient did not feel that the physicians communicated with each other, he or she still answered the follow‐up questions regarding perceptions of physician communication. We excluded this patient from our analysis as his or her answers may not be reliable. The majority of patients (84%) reported this communication as reassuring and felt it was a normal part of procedure performance (94%). Those that did not agree that physician communication was reassuring did not differ in average age (P = 0.307), gender (P = 0.511), or procedure type (P = 0.562).

Physician Communications Measures (N = 64)
 Strongly Agree and Agree No. (%)Neutral No. (%)Disagree, and Strongly Disagree No. (%)
I felt that the physicians talking to each other about my procedure was reassuring to me54 (84)10 (16)0 (0)
Physicians talking to each other while doing a procedure is a normal part of doing a procedure60 (94)4 (6)0 (0)

Of all positive and negative comments, five specifically addressed communication between physicians. Most (four) reflected satisfaction with bedside teaching (eg, They discussed the procedure in a professional manner and eased my mind at all times) and with having an expert in the room (eg, [The team] discussed things like needle placement, which was nice because there was a second opinion right there in the room). Patients also felt that it was good to experience the teaching, with one patient reporting that the best part of the procedure was watching doctors learn from each other. Patients did not express specific reservations about bedside teaching, resident technique, or fear of complications in free text.

Discussion

Even though novice interns performed procedures and simultaneous bedside teaching, patient satisfaction with a teaching procedure service was high, and reported complication rates were low. In addition, a majority of patients found discussions related to teaching activities reassuring and potentially important to their perception of care quality. Analogous studies examining patient satisfaction with endoscopic care found similar rates of patient satisfaction with endoscopists' bedside manner, technical skills, and pain control, but these studies included sedated patients.21 Our results are unique, as we evaluated awake patients with attention to perception of bedside teaching with novice interns.

Our findings offer an alternative strategy for bedside procedural teaching that employs transparency in the use of an expert and a trainee to introduce patients to bedside teaching by experts, which is not common at many academic medical centers.28 Patients may have been reassured by a clear explanation of the role of the service and the providers involved as well as an assurance of expertise and attention to patient comfort and safety. In addition to patient satisfaction, this model has the potential to impact both the safety of bedside procedures and housestaff education around procedure performance. For example, pneumothorax rates using our procedure service model are lower than those published (0% vs. 4% for ultrasound‐guided thoracentesis and 8.5% for thoracentesis by less experienced clinicians).29

Providers may be reluctant to teach at the bedside of awake patients for fear of heightening patient anxiety over trainee inexperience. In the 1960s similar fears were raised over the concern for patient anxiety with bedside rounding,30 but later studies revealed these concerns to be largely unfounded. Instead, bedside rounds have been shown to positively influence patients' feelings about their hospital experience and their relationships with their physicians compared with patients whose case presentations were made in a conference room.31, 32 Given the opportunity to comment on areas for improvement, patients in our study specifically elaborated regarding pain control, communication, and efficiency problems. Although 16% of patients did not find the communication of physicians reassuring, none of the negative comments reflected problems with bedside teaching, but rather concepts such as desiring a better explanation of steps throughout the procedure. Specifically, patients desire better communication for unanticipated pain.

There are several limitations to this study. Lack of patient satisfaction data from a control group of patients whose procedures were performed by attendings or housestaff alone limits our ability to draw conclusions about our satisfaction scores. The scarce applicable literature offers only imperfect comparison data. Because hospitalists were not blinded to the survey, attending behavior may have been subject to a Hawthorne effect.33 Consenting patients after the procedure could have provided hospitalists with an opportunity to exclude patients who appeared less satisfied with their procedure; however, attempts were made to prevent this behavior by requiring strict accounting of why a patient was not consented for the study. Use of alternative personnel for consent such as nurses was explored, but was found not to be feasible due to limited resources. These data are only applicable to English‐speaking patients who are literate and well enough to complete a survey. It is not clear whether the experience for other patients would reflect the same outcomes. It is plausible that non‐English‐speaking patients might have more concerns about incomprehensible conversations taking place during their procedure. Although the surveys were anonymous and patients were told that the proceduralists would not see individual responses, responses may have been biased out of patient concern that their response might affect their care. Hospitalists obtaining consent, however, were careful to stress anonymity and the distinction between the primary team and the procedure team.

Academic hospitals are struggling with providing quality procedural care while balancing housestaff education and experience.28 With hospitalists playing an increasingly prominent role in housestaff education and patient satisfaction initiatives, the supervision of housestaff by trained hospitalist faculty may help meet both aims in the performance of invasive bedside procedures, particularly at institutions where simulation training resources are limited. Although concern may exist for potential patient anxiety with bedside teaching, our data demonstrate high levels of patient satisfaction with a hospitalist procedure service despite novice procedure performers and an emphasis on teaching during the procedure.

References
  1. Smith CC,Gordon CE,Feller‐Kopman D, et al.Creation of an innovative inpatient medical procedure service and a method to evaluate house staff competency.J Gen Intern Med.2004;19(5 Pt 2):510513.
  2. Lucas BP,Asbury JK,Wang Y, et al.Impact of a bedside procedure service on general medicine inpatients: A firm‐based trial.J Hosp Med.2007;2(3):143149.
  3. Hospital Care Quality Information from the Consumer Perspective (HCAHPS).Quality Assurance Guidelines.Baltimore, MD:Centers for Medicare 113(8):657662.
  4. Lehmann LS,Brancati FL,Chen MC,Roter D,Dobs AS.The effect of bedside case presentations on patients' perceptions of their medical care.N Engl J Med.1997;336(16):11501155.
  5. Santen SA,Hemphill RR,Spanier CM,Fletcher ND.‘Sorry, it's my first time!’ Will patients consent to medical students learning procedures?Med Educ.2005;39(4):365369.
  6. Williams CT,Fost N.Ethical considerations surrounding first time procedures: a study and analysis of patient attitudes toward spinal taps by students.Kennedy Inst Ethics J.1992;2(3):217231.
  7. Santen SA,Hemphill RR,McDonald MF,Jo CO.Patients' willingness to allow residents to learn to practice medical procedures.Acad Med.2004;79(2):144147.
  8. Sheets KJ,Caruthers BS,Schwenk TL.Patient satisfaction with gynecologic care provided by family practice resident physicians.Fam Pract Res J.1991;11(4):421428.
  9. Jackson JL,Osgard E,Fincher RK.Resident participation in flexible sigmoidoscopy does not affect patient satisfaction.Am J Gastroenterol.2000;95(6):15631566.
  10. Peterson MA,Abele J.Bedside ultrasound for difficult lumbar puncture.J Emerg Med.2005;28(2):197200.
  11. Grisso T,Applebaum P.Conducting the Assessment. In:Assessing Competence to Consent to Treatment: A Guide for Physicians and Other Health Professionals.First Edition ed.New York, NY:Oxford University Press;1998:8091.
  12. Critchfield JM,Williams MV.Care of Ill, Socially Complicated Patients. In:Medical Management of Vulnerable 2007:407418.
  13. Mueller PR,Biswal S,Halpern EF,Kaufman JA,Lee MJ.Interventional radiologic procedures: patient anxiety, perception of pain, understanding of procedure, and satisfaction with medication‐‐a prospective study.Radiology.2000;215(3):684688.
  14. Hendriks AA,Vrielink MR,Smets EM,van Es SQ,De Haes JC.Improving the assessment of (in)patients' satisfaction with hospital care.Med Care.2001;39(3):270283.
  15. Nguyen Thi PL,Briancon S,Empereur F,Guillemin F.Factors determining inpatient satisfaction with care.Soc Sci Med.2002;54(4):493504.
  16. Hendriks AA,Oort FJ,Vrielink MR,Smets EM.Reliability and validity of the Satisfaction with Hospital Care Questionnaire.Int J Qual Health Care.2002;14(6):471482.
  17. Perneger TV,Kossovsky MP,Cathieni F,di Florio V,Burnand B.A randomized trial of four patient satisfaction questionnaires.Med Care.2003;41(12):13431352.
  18. Gonzalez N,Quintana JM,Bilbao A, et al.Development and validation of an in‐patient satisfaction questionnaire.Int J Qual Health Care.2005;17(6):465472.
  19. Maurer MH,Beck A,Hamm B,Gebauer B.Central venous port catheters: evaluation of patients' satisfaction with implantation under local anesthesia.J Vasc Access.2009;10(1):2732.
  20. Ko HH,Zhang H,Telford JJ,Enns R.Factors influencing patient satisfaction when undergoing endoscopic procedures.Gastrointest Endosc.2009;69(4):88391, quiz 891.e1.
  21. Grogan DR,Irwin RS,Channick R, et al.Complications associated with thoracentesis. A prospective, randomized study comparing three different methods.Arch Intern Med.1990;150(4):873877.
  22. De Gottardi A,Thevenot T,Spahr L, et al.Risk of complications after abdominal paracentesis in cirrhotic patients: a prospective study.Clin Gastroenterol Hepatol.2009;7(8):906909.
  23. Grabau CM,Crago SF,Hoff LK, et al.Performance standards for therapeutic abdominal paracentesis.Hepatology.2004;40(2):484488.
  24. Sempere AP,Berenguer‐Ruiz L,Lezcano‐Rodas M,Mira‐Berenguer F,Waez M.Lumbar puncture: its indications, contraindications, complications and technique.Rev Neurol.2007;45(7):433436.
  25. Allen SH.How to perform a lumbar puncture with the patient in the seated position.Br J Hosp Med (Lond).2006;67(3):M467.
  26. Durning SJ,Cation LJ,Jackson JL.Are commonly used resident measurements associated with procedural skills in internal medicine residency training?J Gen Intern Med.2007;22(3):357361.
  27. Mourad M,Kohlwes J,Maselli J,MERN Group,Auerbach AD.Supervising the Supervisors‐Procedural Training and Supervision in Internal Medicine Residency.J Gen Intern Med.2010.
  28. Gordon CE,Feller‐Kopman D,Balk EM,Smetana GW.Pneumothorax following thoracentesis: a systematic review and meta‐analysis.Arch Intern Med.2010;170(4):332339.
  29. Franzblau AN,Kairys D,Kaufman MR.The emotional impact of ward rounds.J Mt Sinai Hosp NY.1956;23(6):782803.
  30. Simons RJ,Baily RG,Zelis R,Zwillich CW.The physiologic and psychological effects of the bedside presentation.N Engl J Med.1989;321(18):12731275.
  31. Lehmann LS,Brancati FL,Chen MC,Roter D,Dobs AS.The effect of bedside case presentations on patients' perceptions of their medical care.N Engl J Med.1997;336(16):11501155.
  32. Holden JD.Hawthorne effects and research into professional practice.J Eval Clin Pract.2001;7(1):6570.
References
  1. Smith CC,Gordon CE,Feller‐Kopman D, et al.Creation of an innovative inpatient medical procedure service and a method to evaluate house staff competency.J Gen Intern Med.2004;19(5 Pt 2):510513.
  2. Lucas BP,Asbury JK,Wang Y, et al.Impact of a bedside procedure service on general medicine inpatients: A firm‐based trial.J Hosp Med.2007;2(3):143149.
  3. Hospital Care Quality Information from the Consumer Perspective (HCAHPS).Quality Assurance Guidelines.Baltimore, MD:Centers for Medicare 113(8):657662.
  4. Lehmann LS,Brancati FL,Chen MC,Roter D,Dobs AS.The effect of bedside case presentations on patients' perceptions of their medical care.N Engl J Med.1997;336(16):11501155.
  5. Santen SA,Hemphill RR,Spanier CM,Fletcher ND.‘Sorry, it's my first time!’ Will patients consent to medical students learning procedures?Med Educ.2005;39(4):365369.
  6. Williams CT,Fost N.Ethical considerations surrounding first time procedures: a study and analysis of patient attitudes toward spinal taps by students.Kennedy Inst Ethics J.1992;2(3):217231.
  7. Santen SA,Hemphill RR,McDonald MF,Jo CO.Patients' willingness to allow residents to learn to practice medical procedures.Acad Med.2004;79(2):144147.
  8. Sheets KJ,Caruthers BS,Schwenk TL.Patient satisfaction with gynecologic care provided by family practice resident physicians.Fam Pract Res J.1991;11(4):421428.
  9. Jackson JL,Osgard E,Fincher RK.Resident participation in flexible sigmoidoscopy does not affect patient satisfaction.Am J Gastroenterol.2000;95(6):15631566.
  10. Peterson MA,Abele J.Bedside ultrasound for difficult lumbar puncture.J Emerg Med.2005;28(2):197200.
  11. Grisso T,Applebaum P.Conducting the Assessment. In:Assessing Competence to Consent to Treatment: A Guide for Physicians and Other Health Professionals.First Edition ed.New York, NY:Oxford University Press;1998:8091.
  12. Critchfield JM,Williams MV.Care of Ill, Socially Complicated Patients. In:Medical Management of Vulnerable 2007:407418.
  13. Mueller PR,Biswal S,Halpern EF,Kaufman JA,Lee MJ.Interventional radiologic procedures: patient anxiety, perception of pain, understanding of procedure, and satisfaction with medication‐‐a prospective study.Radiology.2000;215(3):684688.
  14. Hendriks AA,Vrielink MR,Smets EM,van Es SQ,De Haes JC.Improving the assessment of (in)patients' satisfaction with hospital care.Med Care.2001;39(3):270283.
  15. Nguyen Thi PL,Briancon S,Empereur F,Guillemin F.Factors determining inpatient satisfaction with care.Soc Sci Med.2002;54(4):493504.
  16. Hendriks AA,Oort FJ,Vrielink MR,Smets EM.Reliability and validity of the Satisfaction with Hospital Care Questionnaire.Int J Qual Health Care.2002;14(6):471482.
  17. Perneger TV,Kossovsky MP,Cathieni F,di Florio V,Burnand B.A randomized trial of four patient satisfaction questionnaires.Med Care.2003;41(12):13431352.
  18. Gonzalez N,Quintana JM,Bilbao A, et al.Development and validation of an in‐patient satisfaction questionnaire.Int J Qual Health Care.2005;17(6):465472.
  19. Maurer MH,Beck A,Hamm B,Gebauer B.Central venous port catheters: evaluation of patients' satisfaction with implantation under local anesthesia.J Vasc Access.2009;10(1):2732.
  20. Ko HH,Zhang H,Telford JJ,Enns R.Factors influencing patient satisfaction when undergoing endoscopic procedures.Gastrointest Endosc.2009;69(4):88391, quiz 891.e1.
  21. Grogan DR,Irwin RS,Channick R, et al.Complications associated with thoracentesis. A prospective, randomized study comparing three different methods.Arch Intern Med.1990;150(4):873877.
  22. De Gottardi A,Thevenot T,Spahr L, et al.Risk of complications after abdominal paracentesis in cirrhotic patients: a prospective study.Clin Gastroenterol Hepatol.2009;7(8):906909.
  23. Grabau CM,Crago SF,Hoff LK, et al.Performance standards for therapeutic abdominal paracentesis.Hepatology.2004;40(2):484488.
  24. Sempere AP,Berenguer‐Ruiz L,Lezcano‐Rodas M,Mira‐Berenguer F,Waez M.Lumbar puncture: its indications, contraindications, complications and technique.Rev Neurol.2007;45(7):433436.
  25. Allen SH.How to perform a lumbar puncture with the patient in the seated position.Br J Hosp Med (Lond).2006;67(3):M467.
  26. Durning SJ,Cation LJ,Jackson JL.Are commonly used resident measurements associated with procedural skills in internal medicine residency training?J Gen Intern Med.2007;22(3):357361.
  27. Mourad M,Kohlwes J,Maselli J,MERN Group,Auerbach AD.Supervising the Supervisors‐Procedural Training and Supervision in Internal Medicine Residency.J Gen Intern Med.2010.
  28. Gordon CE,Feller‐Kopman D,Balk EM,Smetana GW.Pneumothorax following thoracentesis: a systematic review and meta‐analysis.Arch Intern Med.2010;170(4):332339.
  29. Franzblau AN,Kairys D,Kaufman MR.The emotional impact of ward rounds.J Mt Sinai Hosp NY.1956;23(6):782803.
  30. Simons RJ,Baily RG,Zelis R,Zwillich CW.The physiologic and psychological effects of the bedside presentation.N Engl J Med.1989;321(18):12731275.
  31. Lehmann LS,Brancati FL,Chen MC,Roter D,Dobs AS.The effect of bedside case presentations on patients' perceptions of their medical care.N Engl J Med.1997;336(16):11501155.
  32. Holden JD.Hawthorne effects and research into professional practice.J Eval Clin Pract.2001;7(1):6570.
Issue
Journal of Hospital Medicine - 6(4)
Issue
Journal of Hospital Medicine - 6(4)
Page Number
219-224
Page Number
219-224
Publications
Publications
Article Type
Display Headline
Patient satisfaction with a hospitalist procedure service: Is bedside procedure teaching reassuring to patients?
Display Headline
Patient satisfaction with a hospitalist procedure service: Is bedside procedure teaching reassuring to patients?
Legacy Keywords
graduate medical training, patient satisfaction, procedure education, supervision
Legacy Keywords
graduate medical training, patient satisfaction, procedure education, supervision
Sections
Article Source

Copyright © 2011 Society of Hospital Medicine

Disallow All Ads
Correspondence Location
505 Parnassus Ave, M1287, Box 0131, San Francisco, CA 94143
Content Gating
No Gating (article Unlocked/Free)
Alternative CME
Article PDF Media
Media Files

In the Literature

Article Type
Changed
Fri, 09/14/2018 - 12:35
Display Headline
In the Literature

Literature at a Glance

A guide to this month’s studies.

Is Stenting or Endarterectomy Best for Carotid Artery Stenosis?

Background: Patients with moderate to severe symptomatic carotid artery stenosis and those with severe asymptomatic carotid stenosis benefit from carotid endarterectomy. Carotid stenting may provide an alternative therapy, but the long-term protection against stroke compared with endarterectomy is unclear.

Study Design: Prospective randomized trial.

Setting: 29 centers in the United States.

Synopsis: This article reports the long-term (three years) follow-up of the SAPPHIRE trial, published in 2004, which compared carotid stenting to endarterectomy in patients at high surgical risk. In that trial, 334 patients randomized to either stenting or endarterectomy had similar outcomes at one year. Patients were followed for three years with death and major cardiovascular events as endpoints.

Rates of stroke at three years were approximately 10% with an overall death rate of approximately 20%. There was no difference between carotid stenting and endarterectomy with regards to death, stroke, or other cardiovascular outcome.

Notably, follow-up was not complete (78%), a specific type of stenting procedure was used, and the patient population was at high risk for surgical complications. Therefore, results may not be applicable in other centers or in other patient populations. Yet, this trial provides follow-up, long-term evidence that carotid stenting may be a viable alternative to endarterectomy in patients with carotid artery stenosis.

Bottom line: Carotid stenting and endarterectomy had similar outcomes at three years in high-risk patients with carotid artery stenosis.

Citation: Gurm HS, Yadav JS, Fayad P, et al. Long-term results of carotid stenting versus endarterectomy in high-risk patients. N Engl J Med. 2008;358:1572-1579.

Is Early Repolarization on EKG Associated with Sudden Cardiac Arrest?

Background: Electrocardiographic early repolarization, defined as elevation of the QRS-ST junction of at least 0.1mV from baseline in the inferior or lateral leads (manifested as slurring or notching), occurs in 1% to 5% of patients. It is considered benign, but experimental studies have suggested it may be arrhythmogenic.

Study Design: Prospective case-control.

Setting: 22 international tertiary care centers.

Synopsis: Case subjects were less than 60 years of age and were resuscitated after ventricular fibrillation (VF) arrest ultimately deemed idiopathic. All had normal echocardiograms, no evidence of coronary artery disease, and no repolarization abnormalities (including Brugada and long-QT). Of 206 patients, 31% had early repolarization on EKG, versus only 5% in controls without heart disease. In case subjects with prior EKGs, early repolarization was proven to be pre-existing.

The mean magnitude of J-point elevation was 2 mm in cases versus 1.2 mm in controls, and in cases this magnitude increased during later episodes of arrhythmia. Electrophysiologic mapping showed that ectopy originated at sites concordant with the location of abnormal repolarization. During five years of follow-up, arrhythmic recurrence was twice as common in cases with early repolarization.

Although long-term observational studies of persons with early repolarization have shown a benign natural course, this study may change our approach to those with syncope or a family history of sudden death.

 

 

Bottom line: Early repolarization on EKG is associated with idiopathic ventricular fibrillation.

Citation: Haissaguerre M, Derval N, Sacher F, et al. Sudden cardiac arrest associated with early repolarization. N Engl J Med. 2008;358(19):2016-2023.

Does Aggressive Blood Pressure and LDL Treatment in Diabetics Affect Development of Subclinical Atherosclerosis?

Background: There is evidence to suggest more aggressive treatment of LDL cholesterol in patients with known coronary artery disease is beneficial and more aggressive blood pressure control can improve outcomes in some patient populations. However, it is unclear if patients with diabetes without cardiovascular disease would benefit from more aggressive LDL and systolic blood pressure (SBP) treatment.

Study Design: Randomized, open-label, blinded-to-end point trial.

Setting: Four centers in Okla­homa, Arizona, and South Dakota.

Synopsis: Investigators studied 499 type 2 diabetic American Indian men with no history of cardiovascular disease. Patients were randomized to receive treatment to achieve aggressive (70 mg/dL and 115 mmHg) or standard (100 mg/dL and 130 mmHg) targets for their LDL cholesterol and SBP, respectively.

At three years, the aggressive group showed decreased carotid intima-media thickness (IMT) and decreased left ventricular mass, whereas both IMT and left ventricular volume increased in the standard group. There were no differences in clinical cardiovascular events between the aggressive and standard group and both groups had lower-than-expected clinical events.

This study included no women and was limited to an American Indian population. Of note, there was an increase in adverse events related to blood pressure medications in the aggressive group. It also is unclear how the surrogates of cardiovascular disease or subclinical atherosclerosis relate to significant clinical outcomes.

Bottom line: More aggressive LDL and SBP treatment in diabetics without coronary disease decreased subclinical atherosclerosis but did not impact clinical outcomes.

Citation: Howard B, Roman M, Devereux R, et al. Effect of lower targets for blood pressure and LDL cholesterol on atherosclerosis in diabetes. JAMA. 2008;299(14):1678-1689.

Should We Treat Hypertension in Patients Older Than 80?

Background: There is debate about whether treatment of hypertension in the elderly is beneficial. Numerous studies suggest blood pressure control does less to prevent strokes in patients older than 80 years than for younger patients. Moreover, other evidence shows controlling blood pressure in elderly patients may result in an increase in mortality even if there was a decreased risk of stroke.

Study Design: Randomized, double-blind, placebo-controlled trial.

Setting: 195 centers in 13 countries in Europe, China, Australasia, and North Africa.

Synopsis: This study evaluated 3,845 patients, age 80 or older, with a sustained systolic blood pressure (SBP) of 160 mmHg and randomized them to receive indapamide (sustained release) or placebo. Perindopril, or placebo, was added if necessary to achieve a target blood pressure of 150/80 mmHg. Patients who received the indapamide with or without the perindopril had lower blood pressure, lower rate of stroke, lower rate of heart failure, lower rate of death from a cardiovascular cause, and a 21% reduction in all-cause mortality (all statistically significant). There were very few adverse drug events and fewer adverse events overall in the treatment group.

Of note, exclusion criteria included a history of heart failure requiring anti-hypertensive medication, dementia, need for nursing care, an inability to stand or walk, and a creatinine more than 1.7 mg/dL. As well, the “target” SBP of 150 mmHg (which only half of the treatment group achieved) is still considered hypertensive according to the JNC 7 guidelines.

Bottom line: In some patients older than 80, treatment of hypertension may reduce the incidence of stroke, death from stroke, heart failure, and all-cause mortality.

 

 

Citation: Beckett N, Peters R, Fletcher A, et al. Treatment of hypertension in patients 80 years of age or older. N Engl J Med. 2008;358:1887-1898.

What Is the Optimal Hospital LOS for Patients with PE?

Background: Though there are clear trends toward shorter hospital stays after pulmonary embolism (PE), especially with the introduction of low molecular weight heparin, the optimal timing of discharge and the effect of decreased length of stay (LOS) on post-discharge mortality are unknown. Furthermore, there is no risk stratification strategy used to identify low-risk patients with PE who can safely be discharged early or treated in the outpatient setting.

Study Design: Retrospective cohort study.

Setting: 186 acute care hospitals in Pennsylvania from January 2000 to November 2002.

Synopsis: Using a statewide database of 15,531 patients discharged with pulmonary embolism (PE), the authors sought to identify patient and hospital factors associated with LOS and assess whether LOS was associated with post-discharge mortality.

Findings indicate there is considerable variation in LOS for PE between and within hospitals in Pennsylvania. The median LOS for patients with PE was six days; patients with a LOS of four or fewer days had significantly higher post-discharge mortality than patients hospitalized five to eight days. More than half the patients discharged at four or fewer days were classified as high-risk, with Pulmonary Embolism Severity Index (PESI) scores of III-V (3.1% to 24.5% risk of mortality at 30 days).

Although we cannot infer causation (i.e., early discharge=death), clinicians should be aware of the results and consider severity of illness (using PESI or other criteria) in the discharge decision in patients with PE. Future prognostic models and evidence-based criteria would be helpful to identify patients with PE who can be safely discharged early.

Bottom line: Physicians may inappropriately select patients with PE for early discharge who are at increased risk of complications.

Citation: Aujesky D, Stone RA, Kim S, et al. Length of hospital stay and post-discharge mortality in patients with pulmonary embolism. Arch Intern Med. 2008;168(7):706-712.

Do Patients Have a “Good Death” in the Hospital?

Background: Despite an increasing focus on providing appropriate end-of-life care, the majority of patients in developed countries die in the hospital. The circumstances and quality of care provided at the time of death are poorly described.

Study Design: Cross-sectional survey.

Setting: 613 departments in 200 French hospitals.

Synopsis: For 3,793 in-hospital deaths, the investigators surveyed the bedside nurses about the circumstances and details of the death. Twenty-three percent of the patients were admitted for end-of-life care, 29% had a malignancy, and 50% of patients were identified as terminally ill for three days prior to their death.

A family member or relative was present in only 25% of all deaths; 20% of patients were alone at the time of death. In the last few hours of life, up to 70% of patients had symptoms of respiratory distress, while only 44% received opiate analgesia. Only 35% of nurses were satisfied with the quality of death. Satisfaction increased with presence of family members and having written protocols for care at the end of life.

This large, multicenter study has limitations but provides a concerning snapshot of death in the hospital. Hospitalists should be aggressive about symptom control at the end of life as well as attempt to ensure patients are not alone at the time of death.

Bottom line: Many patients die in the hospital in some degree of respiratory distress and without family or friends at the bedside.

Citation: Ferrand E, Jabre P, Vincent-Genod C, et al. Circumstances of death in hospitalized patients and nurses’ perceptions. Arch Intern Med. 2008;168(8):867-875.

 

 

How Common Is Potentially Inappropriate Medication Use in the Hospital?

Background: Use of potentially inappropriate medications (PIM) in the elderly based on the Beers’ List is common in nursing homes, the emergency department (ED), and outpatient settings and is associated with adverse outcomes and hospitalization. Frequency of PIM use the inpatient setting has not been well studied.

Study Design: A retrospective cohort study.

Setting: 384 U.S. hospitals.

Synopsis: In this retrospective cohort study of 493,971 inpatients (older than 65) admitted with medical diagnoses to non-surgeons, PIM prescription was evaluated. Forty nine percent of all patients were prescribed at least one PIM, while 6% were prescribed three or more. In a multivariable model, physician specialty was associated with variation in high severity PIM (HSPIM) prescription. In comparison with internal medicine physicians, cardiologists (odds ratio [OR] 1.32) and pulmonologists (OR 1.10) were more likely to prescribe HSPIMs, while hospitalists (OR 0.90) and geriatricians (OR 0.60) were less likely. In addition, patient age older than 85 was associated with decreased HSPIM prescription (OR 0.59) compared with those younger than 85.

Compared with patients in the Midwest, patients in the South (OR 1.63) and West (OR 1.43) were more likely to prescribe HSPIMs, while those in the Northeast (OR 0.85) were less likely. Hospitals with geriatric services had less PIM use. The study couldn’t account for continuation of chronic medications and did not evaluate adverse outcomes from PIM prescribing.

Bottom line: PIM prescription to hospitalized geriatric patients is common and associated with provider and hospital characteristics.

Citation: Rothberg MB, Pekow PS, Liu F, et al. Potentially inappropriate medication use in hospitalized elders. J Hosp Med. 2008;3:91-102:91-102.

Is There a Benefit to Corticosteroids When Treating Bacterial Meningitis in Children?

Background: The benefit of adjuvant corticosteroids in the treatment of bacterial meningitis in children in the developed world remains unclear; recent expert guidelines reflect this uncertainty.

Study Design: Retrospective cohort study.

Setting: Twenty-seven tertiary care hospitals in the United States.

Synopsis: Researchers examined 2,780 children with a primary diagnosis of bacterial meningitis discharged from 27 tertiary care centers in the U.S. from 2001-2006. Using a propensity analysis (to control for severity of illness), the study compared those who had received adjunctive corticosteroids with those who had not, with mortality and length of study (LOS) as primary outcomes.

The median age was nine months, 8.9% of children received corticosteroids, and the overall mortality rate was 4.2%. Adjuvent corticosteroids did not reduce mortality or LOS. The outcomes were unchanged in subgroup analyses.

Although limited by its retrospective design and lack of other outcome measures (e.g., hearing loss, neurological deficits), this study provides reasonable evidence that corticosteroid use in bacterial meningitis in children may not save lives or shorten LOS. Pediatric hospitalists may not want to routinely give steroids in this setting pending large randomized-controlled trials.

Bottom line: Adjunctive corticosteroids therapy in children with bacterial meningitis may not save lives or reduce LOS.

Citation: Mongelluzzo J, Mohamad Z, Ten Have TR, Shah SS. Corticosteroids and mortality in children with bacterial meningitis. JAMA. 2008;299(17):2048-2055.

Should Unprotected Left Main Disease Be Treated With PCI or CABG?

Background: The current standard of care for the treatment of left main coronary artery disease is coronary-artery bypass grafting (CABG). With the advent of drug-eluting stents, there is growing interest in the use of percutaneous coronary intervention (PCI) to treat left main disease.

Study Design: Prospective observational study.

Setting: Twelve Korean cardiac centers.

Synopsis: From 2000 to 2006, patients with left main disease were treated with PCI or CABG at the discretion of the physician. Nearly 1,100 patients in each cohort were compared and evaluated for death and a composite outcome of death, myocardial infarction, or stroke. Propensity-matching was employed to control for confounders.

 

 

In the overall cohort matched by propensity score, there was no significant difference in death or the composite outcome between the PCI and CABG groups after three years. Type of stent (bare metal vs. drug-eluting) did not affect the outcome. Rates of target-vessel revascularization were significantly higher in the group that received stents.

The results are limited by the observational nature and the need for propensity analysis and yet provide an intriguing result. The standard of care for treatment of left main disease remains CABG, but clinicians may be more comfortable treating with stents while we await randomized-controlled trials.

Bottom line: In this observational study, PCI and CABG had similar outcomes in patients with left main disease.

Citation: Seung KB, Park D, Kim Y, Lee S. Stents versus coronary-artery bypass grafting for left main coronary artery disease. N Engl J Med. 2008;358:1781-1792.

Issue
The Hospitalist - 2008(08)
Publications
Sections

Literature at a Glance

A guide to this month’s studies.

Is Stenting or Endarterectomy Best for Carotid Artery Stenosis?

Background: Patients with moderate to severe symptomatic carotid artery stenosis and those with severe asymptomatic carotid stenosis benefit from carotid endarterectomy. Carotid stenting may provide an alternative therapy, but the long-term protection against stroke compared with endarterectomy is unclear.

Study Design: Prospective randomized trial.

Setting: 29 centers in the United States.

Synopsis: This article reports the long-term (three years) follow-up of the SAPPHIRE trial, published in 2004, which compared carotid stenting to endarterectomy in patients at high surgical risk. In that trial, 334 patients randomized to either stenting or endarterectomy had similar outcomes at one year. Patients were followed for three years with death and major cardiovascular events as endpoints.

Rates of stroke at three years were approximately 10% with an overall death rate of approximately 20%. There was no difference between carotid stenting and endarterectomy with regards to death, stroke, or other cardiovascular outcome.

Notably, follow-up was not complete (78%), a specific type of stenting procedure was used, and the patient population was at high risk for surgical complications. Therefore, results may not be applicable in other centers or in other patient populations. Yet, this trial provides follow-up, long-term evidence that carotid stenting may be a viable alternative to endarterectomy in patients with carotid artery stenosis.

Bottom line: Carotid stenting and endarterectomy had similar outcomes at three years in high-risk patients with carotid artery stenosis.

Citation: Gurm HS, Yadav JS, Fayad P, et al. Long-term results of carotid stenting versus endarterectomy in high-risk patients. N Engl J Med. 2008;358:1572-1579.

Is Early Repolarization on EKG Associated with Sudden Cardiac Arrest?

Background: Electrocardiographic early repolarization, defined as elevation of the QRS-ST junction of at least 0.1mV from baseline in the inferior or lateral leads (manifested as slurring or notching), occurs in 1% to 5% of patients. It is considered benign, but experimental studies have suggested it may be arrhythmogenic.

Study Design: Prospective case-control.

Setting: 22 international tertiary care centers.

Synopsis: Case subjects were less than 60 years of age and were resuscitated after ventricular fibrillation (VF) arrest ultimately deemed idiopathic. All had normal echocardiograms, no evidence of coronary artery disease, and no repolarization abnormalities (including Brugada and long-QT). Of 206 patients, 31% had early repolarization on EKG, versus only 5% in controls without heart disease. In case subjects with prior EKGs, early repolarization was proven to be pre-existing.

The mean magnitude of J-point elevation was 2 mm in cases versus 1.2 mm in controls, and in cases this magnitude increased during later episodes of arrhythmia. Electrophysiologic mapping showed that ectopy originated at sites concordant with the location of abnormal repolarization. During five years of follow-up, arrhythmic recurrence was twice as common in cases with early repolarization.

Although long-term observational studies of persons with early repolarization have shown a benign natural course, this study may change our approach to those with syncope or a family history of sudden death.

 

 

Bottom line: Early repolarization on EKG is associated with idiopathic ventricular fibrillation.

Citation: Haissaguerre M, Derval N, Sacher F, et al. Sudden cardiac arrest associated with early repolarization. N Engl J Med. 2008;358(19):2016-2023.

Does Aggressive Blood Pressure and LDL Treatment in Diabetics Affect Development of Subclinical Atherosclerosis?

Background: There is evidence to suggest more aggressive treatment of LDL cholesterol in patients with known coronary artery disease is beneficial and more aggressive blood pressure control can improve outcomes in some patient populations. However, it is unclear if patients with diabetes without cardiovascular disease would benefit from more aggressive LDL and systolic blood pressure (SBP) treatment.

Study Design: Randomized, open-label, blinded-to-end point trial.

Setting: Four centers in Okla­homa, Arizona, and South Dakota.

Synopsis: Investigators studied 499 type 2 diabetic American Indian men with no history of cardiovascular disease. Patients were randomized to receive treatment to achieve aggressive (70 mg/dL and 115 mmHg) or standard (100 mg/dL and 130 mmHg) targets for their LDL cholesterol and SBP, respectively.

At three years, the aggressive group showed decreased carotid intima-media thickness (IMT) and decreased left ventricular mass, whereas both IMT and left ventricular volume increased in the standard group. There were no differences in clinical cardiovascular events between the aggressive and standard group and both groups had lower-than-expected clinical events.

This study included no women and was limited to an American Indian population. Of note, there was an increase in adverse events related to blood pressure medications in the aggressive group. It also is unclear how the surrogates of cardiovascular disease or subclinical atherosclerosis relate to significant clinical outcomes.

Bottom line: More aggressive LDL and SBP treatment in diabetics without coronary disease decreased subclinical atherosclerosis but did not impact clinical outcomes.

Citation: Howard B, Roman M, Devereux R, et al. Effect of lower targets for blood pressure and LDL cholesterol on atherosclerosis in diabetes. JAMA. 2008;299(14):1678-1689.

Should We Treat Hypertension in Patients Older Than 80?

Background: There is debate about whether treatment of hypertension in the elderly is beneficial. Numerous studies suggest blood pressure control does less to prevent strokes in patients older than 80 years than for younger patients. Moreover, other evidence shows controlling blood pressure in elderly patients may result in an increase in mortality even if there was a decreased risk of stroke.

Study Design: Randomized, double-blind, placebo-controlled trial.

Setting: 195 centers in 13 countries in Europe, China, Australasia, and North Africa.

Synopsis: This study evaluated 3,845 patients, age 80 or older, with a sustained systolic blood pressure (SBP) of 160 mmHg and randomized them to receive indapamide (sustained release) or placebo. Perindopril, or placebo, was added if necessary to achieve a target blood pressure of 150/80 mmHg. Patients who received the indapamide with or without the perindopril had lower blood pressure, lower rate of stroke, lower rate of heart failure, lower rate of death from a cardiovascular cause, and a 21% reduction in all-cause mortality (all statistically significant). There were very few adverse drug events and fewer adverse events overall in the treatment group.

Of note, exclusion criteria included a history of heart failure requiring anti-hypertensive medication, dementia, need for nursing care, an inability to stand or walk, and a creatinine more than 1.7 mg/dL. As well, the “target” SBP of 150 mmHg (which only half of the treatment group achieved) is still considered hypertensive according to the JNC 7 guidelines.

Bottom line: In some patients older than 80, treatment of hypertension may reduce the incidence of stroke, death from stroke, heart failure, and all-cause mortality.

 

 

Citation: Beckett N, Peters R, Fletcher A, et al. Treatment of hypertension in patients 80 years of age or older. N Engl J Med. 2008;358:1887-1898.

What Is the Optimal Hospital LOS for Patients with PE?

Background: Though there are clear trends toward shorter hospital stays after pulmonary embolism (PE), especially with the introduction of low molecular weight heparin, the optimal timing of discharge and the effect of decreased length of stay (LOS) on post-discharge mortality are unknown. Furthermore, there is no risk stratification strategy used to identify low-risk patients with PE who can safely be discharged early or treated in the outpatient setting.

Study Design: Retrospective cohort study.

Setting: 186 acute care hospitals in Pennsylvania from January 2000 to November 2002.

Synopsis: Using a statewide database of 15,531 patients discharged with pulmonary embolism (PE), the authors sought to identify patient and hospital factors associated with LOS and assess whether LOS was associated with post-discharge mortality.

Findings indicate there is considerable variation in LOS for PE between and within hospitals in Pennsylvania. The median LOS for patients with PE was six days; patients with a LOS of four or fewer days had significantly higher post-discharge mortality than patients hospitalized five to eight days. More than half the patients discharged at four or fewer days were classified as high-risk, with Pulmonary Embolism Severity Index (PESI) scores of III-V (3.1% to 24.5% risk of mortality at 30 days).

Although we cannot infer causation (i.e., early discharge=death), clinicians should be aware of the results and consider severity of illness (using PESI or other criteria) in the discharge decision in patients with PE. Future prognostic models and evidence-based criteria would be helpful to identify patients with PE who can be safely discharged early.

Bottom line: Physicians may inappropriately select patients with PE for early discharge who are at increased risk of complications.

Citation: Aujesky D, Stone RA, Kim S, et al. Length of hospital stay and post-discharge mortality in patients with pulmonary embolism. Arch Intern Med. 2008;168(7):706-712.

Do Patients Have a “Good Death” in the Hospital?

Background: Despite an increasing focus on providing appropriate end-of-life care, the majority of patients in developed countries die in the hospital. The circumstances and quality of care provided at the time of death are poorly described.

Study Design: Cross-sectional survey.

Setting: 613 departments in 200 French hospitals.

Synopsis: For 3,793 in-hospital deaths, the investigators surveyed the bedside nurses about the circumstances and details of the death. Twenty-three percent of the patients were admitted for end-of-life care, 29% had a malignancy, and 50% of patients were identified as terminally ill for three days prior to their death.

A family member or relative was present in only 25% of all deaths; 20% of patients were alone at the time of death. In the last few hours of life, up to 70% of patients had symptoms of respiratory distress, while only 44% received opiate analgesia. Only 35% of nurses were satisfied with the quality of death. Satisfaction increased with presence of family members and having written protocols for care at the end of life.

This large, multicenter study has limitations but provides a concerning snapshot of death in the hospital. Hospitalists should be aggressive about symptom control at the end of life as well as attempt to ensure patients are not alone at the time of death.

Bottom line: Many patients die in the hospital in some degree of respiratory distress and without family or friends at the bedside.

Citation: Ferrand E, Jabre P, Vincent-Genod C, et al. Circumstances of death in hospitalized patients and nurses’ perceptions. Arch Intern Med. 2008;168(8):867-875.

 

 

How Common Is Potentially Inappropriate Medication Use in the Hospital?

Background: Use of potentially inappropriate medications (PIM) in the elderly based on the Beers’ List is common in nursing homes, the emergency department (ED), and outpatient settings and is associated with adverse outcomes and hospitalization. Frequency of PIM use the inpatient setting has not been well studied.

Study Design: A retrospective cohort study.

Setting: 384 U.S. hospitals.

Synopsis: In this retrospective cohort study of 493,971 inpatients (older than 65) admitted with medical diagnoses to non-surgeons, PIM prescription was evaluated. Forty nine percent of all patients were prescribed at least one PIM, while 6% were prescribed three or more. In a multivariable model, physician specialty was associated with variation in high severity PIM (HSPIM) prescription. In comparison with internal medicine physicians, cardiologists (odds ratio [OR] 1.32) and pulmonologists (OR 1.10) were more likely to prescribe HSPIMs, while hospitalists (OR 0.90) and geriatricians (OR 0.60) were less likely. In addition, patient age older than 85 was associated with decreased HSPIM prescription (OR 0.59) compared with those younger than 85.

Compared with patients in the Midwest, patients in the South (OR 1.63) and West (OR 1.43) were more likely to prescribe HSPIMs, while those in the Northeast (OR 0.85) were less likely. Hospitals with geriatric services had less PIM use. The study couldn’t account for continuation of chronic medications and did not evaluate adverse outcomes from PIM prescribing.

Bottom line: PIM prescription to hospitalized geriatric patients is common and associated with provider and hospital characteristics.

Citation: Rothberg MB, Pekow PS, Liu F, et al. Potentially inappropriate medication use in hospitalized elders. J Hosp Med. 2008;3:91-102:91-102.

Is There a Benefit to Corticosteroids When Treating Bacterial Meningitis in Children?

Background: The benefit of adjuvant corticosteroids in the treatment of bacterial meningitis in children in the developed world remains unclear; recent expert guidelines reflect this uncertainty.

Study Design: Retrospective cohort study.

Setting: Twenty-seven tertiary care hospitals in the United States.

Synopsis: Researchers examined 2,780 children with a primary diagnosis of bacterial meningitis discharged from 27 tertiary care centers in the U.S. from 2001-2006. Using a propensity analysis (to control for severity of illness), the study compared those who had received adjunctive corticosteroids with those who had not, with mortality and length of study (LOS) as primary outcomes.

The median age was nine months, 8.9% of children received corticosteroids, and the overall mortality rate was 4.2%. Adjuvent corticosteroids did not reduce mortality or LOS. The outcomes were unchanged in subgroup analyses.

Although limited by its retrospective design and lack of other outcome measures (e.g., hearing loss, neurological deficits), this study provides reasonable evidence that corticosteroid use in bacterial meningitis in children may not save lives or shorten LOS. Pediatric hospitalists may not want to routinely give steroids in this setting pending large randomized-controlled trials.

Bottom line: Adjunctive corticosteroids therapy in children with bacterial meningitis may not save lives or reduce LOS.

Citation: Mongelluzzo J, Mohamad Z, Ten Have TR, Shah SS. Corticosteroids and mortality in children with bacterial meningitis. JAMA. 2008;299(17):2048-2055.

Should Unprotected Left Main Disease Be Treated With PCI or CABG?

Background: The current standard of care for the treatment of left main coronary artery disease is coronary-artery bypass grafting (CABG). With the advent of drug-eluting stents, there is growing interest in the use of percutaneous coronary intervention (PCI) to treat left main disease.

Study Design: Prospective observational study.

Setting: Twelve Korean cardiac centers.

Synopsis: From 2000 to 2006, patients with left main disease were treated with PCI or CABG at the discretion of the physician. Nearly 1,100 patients in each cohort were compared and evaluated for death and a composite outcome of death, myocardial infarction, or stroke. Propensity-matching was employed to control for confounders.

 

 

In the overall cohort matched by propensity score, there was no significant difference in death or the composite outcome between the PCI and CABG groups after three years. Type of stent (bare metal vs. drug-eluting) did not affect the outcome. Rates of target-vessel revascularization were significantly higher in the group that received stents.

The results are limited by the observational nature and the need for propensity analysis and yet provide an intriguing result. The standard of care for treatment of left main disease remains CABG, but clinicians may be more comfortable treating with stents while we await randomized-controlled trials.

Bottom line: In this observational study, PCI and CABG had similar outcomes in patients with left main disease.

Citation: Seung KB, Park D, Kim Y, Lee S. Stents versus coronary-artery bypass grafting for left main coronary artery disease. N Engl J Med. 2008;358:1781-1792.

Literature at a Glance

A guide to this month’s studies.

Is Stenting or Endarterectomy Best for Carotid Artery Stenosis?

Background: Patients with moderate to severe symptomatic carotid artery stenosis and those with severe asymptomatic carotid stenosis benefit from carotid endarterectomy. Carotid stenting may provide an alternative therapy, but the long-term protection against stroke compared with endarterectomy is unclear.

Study Design: Prospective randomized trial.

Setting: 29 centers in the United States.

Synopsis: This article reports the long-term (three years) follow-up of the SAPPHIRE trial, published in 2004, which compared carotid stenting to endarterectomy in patients at high surgical risk. In that trial, 334 patients randomized to either stenting or endarterectomy had similar outcomes at one year. Patients were followed for three years with death and major cardiovascular events as endpoints.

Rates of stroke at three years were approximately 10% with an overall death rate of approximately 20%. There was no difference between carotid stenting and endarterectomy with regards to death, stroke, or other cardiovascular outcome.

Notably, follow-up was not complete (78%), a specific type of stenting procedure was used, and the patient population was at high risk for surgical complications. Therefore, results may not be applicable in other centers or in other patient populations. Yet, this trial provides follow-up, long-term evidence that carotid stenting may be a viable alternative to endarterectomy in patients with carotid artery stenosis.

Bottom line: Carotid stenting and endarterectomy had similar outcomes at three years in high-risk patients with carotid artery stenosis.

Citation: Gurm HS, Yadav JS, Fayad P, et al. Long-term results of carotid stenting versus endarterectomy in high-risk patients. N Engl J Med. 2008;358:1572-1579.

Is Early Repolarization on EKG Associated with Sudden Cardiac Arrest?

Background: Electrocardiographic early repolarization, defined as elevation of the QRS-ST junction of at least 0.1mV from baseline in the inferior or lateral leads (manifested as slurring or notching), occurs in 1% to 5% of patients. It is considered benign, but experimental studies have suggested it may be arrhythmogenic.

Study Design: Prospective case-control.

Setting: 22 international tertiary care centers.

Synopsis: Case subjects were less than 60 years of age and were resuscitated after ventricular fibrillation (VF) arrest ultimately deemed idiopathic. All had normal echocardiograms, no evidence of coronary artery disease, and no repolarization abnormalities (including Brugada and long-QT). Of 206 patients, 31% had early repolarization on EKG, versus only 5% in controls without heart disease. In case subjects with prior EKGs, early repolarization was proven to be pre-existing.

The mean magnitude of J-point elevation was 2 mm in cases versus 1.2 mm in controls, and in cases this magnitude increased during later episodes of arrhythmia. Electrophysiologic mapping showed that ectopy originated at sites concordant with the location of abnormal repolarization. During five years of follow-up, arrhythmic recurrence was twice as common in cases with early repolarization.

Although long-term observational studies of persons with early repolarization have shown a benign natural course, this study may change our approach to those with syncope or a family history of sudden death.

 

 

Bottom line: Early repolarization on EKG is associated with idiopathic ventricular fibrillation.

Citation: Haissaguerre M, Derval N, Sacher F, et al. Sudden cardiac arrest associated with early repolarization. N Engl J Med. 2008;358(19):2016-2023.

Does Aggressive Blood Pressure and LDL Treatment in Diabetics Affect Development of Subclinical Atherosclerosis?

Background: There is evidence to suggest more aggressive treatment of LDL cholesterol in patients with known coronary artery disease is beneficial and more aggressive blood pressure control can improve outcomes in some patient populations. However, it is unclear if patients with diabetes without cardiovascular disease would benefit from more aggressive LDL and systolic blood pressure (SBP) treatment.

Study Design: Randomized, open-label, blinded-to-end point trial.

Setting: Four centers in Okla­homa, Arizona, and South Dakota.

Synopsis: Investigators studied 499 type 2 diabetic American Indian men with no history of cardiovascular disease. Patients were randomized to receive treatment to achieve aggressive (70 mg/dL and 115 mmHg) or standard (100 mg/dL and 130 mmHg) targets for their LDL cholesterol and SBP, respectively.

At three years, the aggressive group showed decreased carotid intima-media thickness (IMT) and decreased left ventricular mass, whereas both IMT and left ventricular volume increased in the standard group. There were no differences in clinical cardiovascular events between the aggressive and standard group and both groups had lower-than-expected clinical events.

This study included no women and was limited to an American Indian population. Of note, there was an increase in adverse events related to blood pressure medications in the aggressive group. It also is unclear how the surrogates of cardiovascular disease or subclinical atherosclerosis relate to significant clinical outcomes.

Bottom line: More aggressive LDL and SBP treatment in diabetics without coronary disease decreased subclinical atherosclerosis but did not impact clinical outcomes.

Citation: Howard B, Roman M, Devereux R, et al. Effect of lower targets for blood pressure and LDL cholesterol on atherosclerosis in diabetes. JAMA. 2008;299(14):1678-1689.

Should We Treat Hypertension in Patients Older Than 80?

Background: There is debate about whether treatment of hypertension in the elderly is beneficial. Numerous studies suggest blood pressure control does less to prevent strokes in patients older than 80 years than for younger patients. Moreover, other evidence shows controlling blood pressure in elderly patients may result in an increase in mortality even if there was a decreased risk of stroke.

Study Design: Randomized, double-blind, placebo-controlled trial.

Setting: 195 centers in 13 countries in Europe, China, Australasia, and North Africa.

Synopsis: This study evaluated 3,845 patients, age 80 or older, with a sustained systolic blood pressure (SBP) of 160 mmHg and randomized them to receive indapamide (sustained release) or placebo. Perindopril, or placebo, was added if necessary to achieve a target blood pressure of 150/80 mmHg. Patients who received the indapamide with or without the perindopril had lower blood pressure, lower rate of stroke, lower rate of heart failure, lower rate of death from a cardiovascular cause, and a 21% reduction in all-cause mortality (all statistically significant). There were very few adverse drug events and fewer adverse events overall in the treatment group.

Of note, exclusion criteria included a history of heart failure requiring anti-hypertensive medication, dementia, need for nursing care, an inability to stand or walk, and a creatinine more than 1.7 mg/dL. As well, the “target” SBP of 150 mmHg (which only half of the treatment group achieved) is still considered hypertensive according to the JNC 7 guidelines.

Bottom line: In some patients older than 80, treatment of hypertension may reduce the incidence of stroke, death from stroke, heart failure, and all-cause mortality.

 

 

Citation: Beckett N, Peters R, Fletcher A, et al. Treatment of hypertension in patients 80 years of age or older. N Engl J Med. 2008;358:1887-1898.

What Is the Optimal Hospital LOS for Patients with PE?

Background: Though there are clear trends toward shorter hospital stays after pulmonary embolism (PE), especially with the introduction of low molecular weight heparin, the optimal timing of discharge and the effect of decreased length of stay (LOS) on post-discharge mortality are unknown. Furthermore, there is no risk stratification strategy used to identify low-risk patients with PE who can safely be discharged early or treated in the outpatient setting.

Study Design: Retrospective cohort study.

Setting: 186 acute care hospitals in Pennsylvania from January 2000 to November 2002.

Synopsis: Using a statewide database of 15,531 patients discharged with pulmonary embolism (PE), the authors sought to identify patient and hospital factors associated with LOS and assess whether LOS was associated with post-discharge mortality.

Findings indicate there is considerable variation in LOS for PE between and within hospitals in Pennsylvania. The median LOS for patients with PE was six days; patients with a LOS of four or fewer days had significantly higher post-discharge mortality than patients hospitalized five to eight days. More than half the patients discharged at four or fewer days were classified as high-risk, with Pulmonary Embolism Severity Index (PESI) scores of III-V (3.1% to 24.5% risk of mortality at 30 days).

Although we cannot infer causation (i.e., early discharge=death), clinicians should be aware of the results and consider severity of illness (using PESI or other criteria) in the discharge decision in patients with PE. Future prognostic models and evidence-based criteria would be helpful to identify patients with PE who can be safely discharged early.

Bottom line: Physicians may inappropriately select patients with PE for early discharge who are at increased risk of complications.

Citation: Aujesky D, Stone RA, Kim S, et al. Length of hospital stay and post-discharge mortality in patients with pulmonary embolism. Arch Intern Med. 2008;168(7):706-712.

Do Patients Have a “Good Death” in the Hospital?

Background: Despite an increasing focus on providing appropriate end-of-life care, the majority of patients in developed countries die in the hospital. The circumstances and quality of care provided at the time of death are poorly described.

Study Design: Cross-sectional survey.

Setting: 613 departments in 200 French hospitals.

Synopsis: For 3,793 in-hospital deaths, the investigators surveyed the bedside nurses about the circumstances and details of the death. Twenty-three percent of the patients were admitted for end-of-life care, 29% had a malignancy, and 50% of patients were identified as terminally ill for three days prior to their death.

A family member or relative was present in only 25% of all deaths; 20% of patients were alone at the time of death. In the last few hours of life, up to 70% of patients had symptoms of respiratory distress, while only 44% received opiate analgesia. Only 35% of nurses were satisfied with the quality of death. Satisfaction increased with presence of family members and having written protocols for care at the end of life.

This large, multicenter study has limitations but provides a concerning snapshot of death in the hospital. Hospitalists should be aggressive about symptom control at the end of life as well as attempt to ensure patients are not alone at the time of death.

Bottom line: Many patients die in the hospital in some degree of respiratory distress and without family or friends at the bedside.

Citation: Ferrand E, Jabre P, Vincent-Genod C, et al. Circumstances of death in hospitalized patients and nurses’ perceptions. Arch Intern Med. 2008;168(8):867-875.

 

 

How Common Is Potentially Inappropriate Medication Use in the Hospital?

Background: Use of potentially inappropriate medications (PIM) in the elderly based on the Beers’ List is common in nursing homes, the emergency department (ED), and outpatient settings and is associated with adverse outcomes and hospitalization. Frequency of PIM use the inpatient setting has not been well studied.

Study Design: A retrospective cohort study.

Setting: 384 U.S. hospitals.

Synopsis: In this retrospective cohort study of 493,971 inpatients (older than 65) admitted with medical diagnoses to non-surgeons, PIM prescription was evaluated. Forty nine percent of all patients were prescribed at least one PIM, while 6% were prescribed three or more. In a multivariable model, physician specialty was associated with variation in high severity PIM (HSPIM) prescription. In comparison with internal medicine physicians, cardiologists (odds ratio [OR] 1.32) and pulmonologists (OR 1.10) were more likely to prescribe HSPIMs, while hospitalists (OR 0.90) and geriatricians (OR 0.60) were less likely. In addition, patient age older than 85 was associated with decreased HSPIM prescription (OR 0.59) compared with those younger than 85.

Compared with patients in the Midwest, patients in the South (OR 1.63) and West (OR 1.43) were more likely to prescribe HSPIMs, while those in the Northeast (OR 0.85) were less likely. Hospitals with geriatric services had less PIM use. The study couldn’t account for continuation of chronic medications and did not evaluate adverse outcomes from PIM prescribing.

Bottom line: PIM prescription to hospitalized geriatric patients is common and associated with provider and hospital characteristics.

Citation: Rothberg MB, Pekow PS, Liu F, et al. Potentially inappropriate medication use in hospitalized elders. J Hosp Med. 2008;3:91-102:91-102.

Is There a Benefit to Corticosteroids When Treating Bacterial Meningitis in Children?

Background: The benefit of adjuvant corticosteroids in the treatment of bacterial meningitis in children in the developed world remains unclear; recent expert guidelines reflect this uncertainty.

Study Design: Retrospective cohort study.

Setting: Twenty-seven tertiary care hospitals in the United States.

Synopsis: Researchers examined 2,780 children with a primary diagnosis of bacterial meningitis discharged from 27 tertiary care centers in the U.S. from 2001-2006. Using a propensity analysis (to control for severity of illness), the study compared those who had received adjunctive corticosteroids with those who had not, with mortality and length of study (LOS) as primary outcomes.

The median age was nine months, 8.9% of children received corticosteroids, and the overall mortality rate was 4.2%. Adjuvent corticosteroids did not reduce mortality or LOS. The outcomes were unchanged in subgroup analyses.

Although limited by its retrospective design and lack of other outcome measures (e.g., hearing loss, neurological deficits), this study provides reasonable evidence that corticosteroid use in bacterial meningitis in children may not save lives or shorten LOS. Pediatric hospitalists may not want to routinely give steroids in this setting pending large randomized-controlled trials.

Bottom line: Adjunctive corticosteroids therapy in children with bacterial meningitis may not save lives or reduce LOS.

Citation: Mongelluzzo J, Mohamad Z, Ten Have TR, Shah SS. Corticosteroids and mortality in children with bacterial meningitis. JAMA. 2008;299(17):2048-2055.

Should Unprotected Left Main Disease Be Treated With PCI or CABG?

Background: The current standard of care for the treatment of left main coronary artery disease is coronary-artery bypass grafting (CABG). With the advent of drug-eluting stents, there is growing interest in the use of percutaneous coronary intervention (PCI) to treat left main disease.

Study Design: Prospective observational study.

Setting: Twelve Korean cardiac centers.

Synopsis: From 2000 to 2006, patients with left main disease were treated with PCI or CABG at the discretion of the physician. Nearly 1,100 patients in each cohort were compared and evaluated for death and a composite outcome of death, myocardial infarction, or stroke. Propensity-matching was employed to control for confounders.

 

 

In the overall cohort matched by propensity score, there was no significant difference in death or the composite outcome between the PCI and CABG groups after three years. Type of stent (bare metal vs. drug-eluting) did not affect the outcome. Rates of target-vessel revascularization were significantly higher in the group that received stents.

The results are limited by the observational nature and the need for propensity analysis and yet provide an intriguing result. The standard of care for treatment of left main disease remains CABG, but clinicians may be more comfortable treating with stents while we await randomized-controlled trials.

Bottom line: In this observational study, PCI and CABG had similar outcomes in patients with left main disease.

Citation: Seung KB, Park D, Kim Y, Lee S. Stents versus coronary-artery bypass grafting for left main coronary artery disease. N Engl J Med. 2008;358:1781-1792.

Issue
The Hospitalist - 2008(08)
Issue
The Hospitalist - 2008(08)
Publications
Publications
Article Type
Display Headline
In the Literature
Display Headline
In the Literature
Sections
Disallow All Ads
Content Gating
No Gating (article Unlocked/Free)

Your First Job

Article Type
Changed
Fri, 09/14/2018 - 12:37
Display Headline
Your First Job

Within the next few months, many of you will have a new job as an attending hospitalist. As daunting as that may seem, now is the time to think about what you can do to ensure a smooth transition and successful beginning to your career.

Although residency prepared you to face the medical challenges ahead, here are 10 pointers that may help as you move to the next stage of your professional life.

CONTRACT PRIMER

Prior to committing to your first job out of residency, review the contract with a keen eye. Get the advice of a healthcare lawyer before signing a contract and agreeing to the stated conditions. This can prove invaluable especially during contract negotiations. Pay particular attention to the following items:

Malpractice Policies

Identify the type of malpractice coverage your employer provides. The types include:

  • Occurrence malpractice insurance: Provides coverage of claims regardless of when the claim was filed for incidents occurring during the policy period. This insurance will offer seamless coverage even in the event of an insurance policy or job change.
  • Claims-made insurance: Provides coverage of a claim only if the incident occurred and the claim was filed during the policy period. If your claims-made policy is canceled or expired, then coverage no longer exists for any filed claims. “Tail” coverage (liability insurance) is needed at this point.
  • Liability insurance: Provides continued coverage against claims after your claims-made policy has ended. It can be expensive, but prevents gaps in coverage and protects the physician in the event of litigation.

If your contract states you will be provided a claims-made malpractice policy, get details about your liability insurance in writing. Find out how much liability insurance costs, who pays for it, and what affects the cost if you must pay for it. These variables can include tenure with the group as well as circumstances for departure from the group.

Restrictive Covenants

Also called noncompete clauses, these prevent hospitalists from practicing within a geographic region of a previous employer for a certain amount of time. Most noncompete clauses are enforceable by law.

Understand the terms of any restrictive covenant and make sure you’re comfortable with them. Carefully assess any geographic restrictions and how long you’ll remain subject to them. Given the dynamic environment of hospital medicine and the growing demand for hospitalists, these clauses are especially vital.

1) Familiarize yourself with the licensing/credentialing process: Do not underestimate the amount of time it takes to get this paperwork approved—up to six months in some cases. Many new hires’ first days on the job are delayed because they didn’t complete this step. Check with state licensing boards for special requirements unique to that state. Also, every hospital has its own gauntlet of infectious disease, HIPAA, and information- technology hoops to jump through. Getting your applications in as early as possible puts you in position to begin on your planned start date and prevent last-minute catastrophes for your new program.

2) Gain valuable insight through observation: Study your current hospitalist group to gain perspective that will help in your new setting. All programs and hospitals operate differently and have room for quality/process improvement. Interview hospitalists, ask questions, and observe the workflow in your current hospital(s) to help in your new job.

3) Contemplate your career direction: Think strategically about your strengths and plans. Although you will learn an incredible amount about hospital medicine careers after you begin, having a sense of direction will help your new group and its leadership get you where you want to be. Making connections and making your goals known within your new program before you start will put your new career on the right path.

 

 

4) Seek mentors: Having mentors from your prior program and your new program is a key to a healthy and happy career. Choose people you respect and pick their brains about their careers, how they acquired their skills, and how they would advise you to do the same. Good mentors will help you for many years, and the most valuable may be the ones who have known you throughout your residency. Nurture and maintain these relationships even if you are moving on to new horizons. Inquire whether your new program has a mentorship structure or if your new group leader can recommend someone who shares common interests and goals.

5) Study SHM’s Core Com-petencies: Although you may have trepidation about your medical skills and knowledge as you move into uncharted waters, step back and relax. Know that you are prepared. That said, you can always learn more. One excellent resource is The Core Competencies in Hospital Medicine: A Framework for Curriculum Development (available on SHM’s Web site, www.hospitalmedicine.org). This is a set of standards with which programs can teach hospital medicine and you can learn the scope of expectations and competencies for someone in your position.

6) Understand the nuts and bolts of your new program: Although there are many things you will learn on the job, gain an appreciation for some of the following before your first day:

  • Billing: If this is your responsibility, you need to learn a little about this before you start, preferably from one of your future colleagues.
  • Reimbursement structure: Find out how your productivity is tracked and rewarded. You’d be amazed how variable this can be.
  • Time allotment: How are administrative, research, committee and teaching time balanced against your clinical time?

7) Get to know your new hospital: Before hitting the wards it pays to do a little homework on your new workplace. Do you have access to a medical library, journals, UpToDate, or other online databases? If not, do you need to purchase this access on your own? Many programs have academic funds allotted so you can use those resources. Also, familiarize yourself with the local antibiogram, formularies, guidelines, and order sets. Most facilities have tools specific to their hospital. Know how these affect you in your new role. Prior to starting, you will also want to be sufficiently oriented to any computer systems and understand how they’re used for documentation and order entry, and for viewing lab, radiology, and microbiology results.

8) Shadow a hospitalist: Spending a few hours with someone during a typical hospitalist work day will give you an idea of the pace of the work, the layout of the hospital and floors, the medical and ancillary staff you will work with, and the patient population. This will prompt questions you hadn’t thought of previously.

9) Prepare for each specific role: Hospitalists wear many hats, including teaching attending, non-teaching attending, consultant, researcher, committee member, and hospital medicine leader. Each role carries specific responsibilities and expectations. Prior to each new role, train with someone who leads that service or knows the job intimately.

10) Comprehend your benefits: Does your employer have a retirement program? Do they match retirement contributions? How does the malpractice insurance work? A meeting with human resources will usually help you arrange your health, dental, malpractice, and disability insurance prior to your start date. TH

Dr. Chacko is chair of SHM’s young physician committee and the hospitalist program medical director for Preferred Health Partners in New York City. Dr. Markoff is an assistant professor of medicine and associate director of the hospitalist service at the Mount Sinai Hospital in New York City. Dr. Sliwka is a hospitalist and assistant professor of clinical medicine at the University of California, San Francisco Medical Center.

Issue
The Hospitalist - 2008(01)
Publications
Sections

Within the next few months, many of you will have a new job as an attending hospitalist. As daunting as that may seem, now is the time to think about what you can do to ensure a smooth transition and successful beginning to your career.

Although residency prepared you to face the medical challenges ahead, here are 10 pointers that may help as you move to the next stage of your professional life.

CONTRACT PRIMER

Prior to committing to your first job out of residency, review the contract with a keen eye. Get the advice of a healthcare lawyer before signing a contract and agreeing to the stated conditions. This can prove invaluable especially during contract negotiations. Pay particular attention to the following items:

Malpractice Policies

Identify the type of malpractice coverage your employer provides. The types include:

  • Occurrence malpractice insurance: Provides coverage of claims regardless of when the claim was filed for incidents occurring during the policy period. This insurance will offer seamless coverage even in the event of an insurance policy or job change.
  • Claims-made insurance: Provides coverage of a claim only if the incident occurred and the claim was filed during the policy period. If your claims-made policy is canceled or expired, then coverage no longer exists for any filed claims. “Tail” coverage (liability insurance) is needed at this point.
  • Liability insurance: Provides continued coverage against claims after your claims-made policy has ended. It can be expensive, but prevents gaps in coverage and protects the physician in the event of litigation.

If your contract states you will be provided a claims-made malpractice policy, get details about your liability insurance in writing. Find out how much liability insurance costs, who pays for it, and what affects the cost if you must pay for it. These variables can include tenure with the group as well as circumstances for departure from the group.

Restrictive Covenants

Also called noncompete clauses, these prevent hospitalists from practicing within a geographic region of a previous employer for a certain amount of time. Most noncompete clauses are enforceable by law.

Understand the terms of any restrictive covenant and make sure you’re comfortable with them. Carefully assess any geographic restrictions and how long you’ll remain subject to them. Given the dynamic environment of hospital medicine and the growing demand for hospitalists, these clauses are especially vital.

1) Familiarize yourself with the licensing/credentialing process: Do not underestimate the amount of time it takes to get this paperwork approved—up to six months in some cases. Many new hires’ first days on the job are delayed because they didn’t complete this step. Check with state licensing boards for special requirements unique to that state. Also, every hospital has its own gauntlet of infectious disease, HIPAA, and information- technology hoops to jump through. Getting your applications in as early as possible puts you in position to begin on your planned start date and prevent last-minute catastrophes for your new program.

2) Gain valuable insight through observation: Study your current hospitalist group to gain perspective that will help in your new setting. All programs and hospitals operate differently and have room for quality/process improvement. Interview hospitalists, ask questions, and observe the workflow in your current hospital(s) to help in your new job.

3) Contemplate your career direction: Think strategically about your strengths and plans. Although you will learn an incredible amount about hospital medicine careers after you begin, having a sense of direction will help your new group and its leadership get you where you want to be. Making connections and making your goals known within your new program before you start will put your new career on the right path.

 

 

4) Seek mentors: Having mentors from your prior program and your new program is a key to a healthy and happy career. Choose people you respect and pick their brains about their careers, how they acquired their skills, and how they would advise you to do the same. Good mentors will help you for many years, and the most valuable may be the ones who have known you throughout your residency. Nurture and maintain these relationships even if you are moving on to new horizons. Inquire whether your new program has a mentorship structure or if your new group leader can recommend someone who shares common interests and goals.

5) Study SHM’s Core Com-petencies: Although you may have trepidation about your medical skills and knowledge as you move into uncharted waters, step back and relax. Know that you are prepared. That said, you can always learn more. One excellent resource is The Core Competencies in Hospital Medicine: A Framework for Curriculum Development (available on SHM’s Web site, www.hospitalmedicine.org). This is a set of standards with which programs can teach hospital medicine and you can learn the scope of expectations and competencies for someone in your position.

6) Understand the nuts and bolts of your new program: Although there are many things you will learn on the job, gain an appreciation for some of the following before your first day:

  • Billing: If this is your responsibility, you need to learn a little about this before you start, preferably from one of your future colleagues.
  • Reimbursement structure: Find out how your productivity is tracked and rewarded. You’d be amazed how variable this can be.
  • Time allotment: How are administrative, research, committee and teaching time balanced against your clinical time?

7) Get to know your new hospital: Before hitting the wards it pays to do a little homework on your new workplace. Do you have access to a medical library, journals, UpToDate, or other online databases? If not, do you need to purchase this access on your own? Many programs have academic funds allotted so you can use those resources. Also, familiarize yourself with the local antibiogram, formularies, guidelines, and order sets. Most facilities have tools specific to their hospital. Know how these affect you in your new role. Prior to starting, you will also want to be sufficiently oriented to any computer systems and understand how they’re used for documentation and order entry, and for viewing lab, radiology, and microbiology results.

8) Shadow a hospitalist: Spending a few hours with someone during a typical hospitalist work day will give you an idea of the pace of the work, the layout of the hospital and floors, the medical and ancillary staff you will work with, and the patient population. This will prompt questions you hadn’t thought of previously.

9) Prepare for each specific role: Hospitalists wear many hats, including teaching attending, non-teaching attending, consultant, researcher, committee member, and hospital medicine leader. Each role carries specific responsibilities and expectations. Prior to each new role, train with someone who leads that service or knows the job intimately.

10) Comprehend your benefits: Does your employer have a retirement program? Do they match retirement contributions? How does the malpractice insurance work? A meeting with human resources will usually help you arrange your health, dental, malpractice, and disability insurance prior to your start date. TH

Dr. Chacko is chair of SHM’s young physician committee and the hospitalist program medical director for Preferred Health Partners in New York City. Dr. Markoff is an assistant professor of medicine and associate director of the hospitalist service at the Mount Sinai Hospital in New York City. Dr. Sliwka is a hospitalist and assistant professor of clinical medicine at the University of California, San Francisco Medical Center.

Within the next few months, many of you will have a new job as an attending hospitalist. As daunting as that may seem, now is the time to think about what you can do to ensure a smooth transition and successful beginning to your career.

Although residency prepared you to face the medical challenges ahead, here are 10 pointers that may help as you move to the next stage of your professional life.

CONTRACT PRIMER

Prior to committing to your first job out of residency, review the contract with a keen eye. Get the advice of a healthcare lawyer before signing a contract and agreeing to the stated conditions. This can prove invaluable especially during contract negotiations. Pay particular attention to the following items:

Malpractice Policies

Identify the type of malpractice coverage your employer provides. The types include:

  • Occurrence malpractice insurance: Provides coverage of claims regardless of when the claim was filed for incidents occurring during the policy period. This insurance will offer seamless coverage even in the event of an insurance policy or job change.
  • Claims-made insurance: Provides coverage of a claim only if the incident occurred and the claim was filed during the policy period. If your claims-made policy is canceled or expired, then coverage no longer exists for any filed claims. “Tail” coverage (liability insurance) is needed at this point.
  • Liability insurance: Provides continued coverage against claims after your claims-made policy has ended. It can be expensive, but prevents gaps in coverage and protects the physician in the event of litigation.

If your contract states you will be provided a claims-made malpractice policy, get details about your liability insurance in writing. Find out how much liability insurance costs, who pays for it, and what affects the cost if you must pay for it. These variables can include tenure with the group as well as circumstances for departure from the group.

Restrictive Covenants

Also called noncompete clauses, these prevent hospitalists from practicing within a geographic region of a previous employer for a certain amount of time. Most noncompete clauses are enforceable by law.

Understand the terms of any restrictive covenant and make sure you’re comfortable with them. Carefully assess any geographic restrictions and how long you’ll remain subject to them. Given the dynamic environment of hospital medicine and the growing demand for hospitalists, these clauses are especially vital.

1) Familiarize yourself with the licensing/credentialing process: Do not underestimate the amount of time it takes to get this paperwork approved—up to six months in some cases. Many new hires’ first days on the job are delayed because they didn’t complete this step. Check with state licensing boards for special requirements unique to that state. Also, every hospital has its own gauntlet of infectious disease, HIPAA, and information- technology hoops to jump through. Getting your applications in as early as possible puts you in position to begin on your planned start date and prevent last-minute catastrophes for your new program.

2) Gain valuable insight through observation: Study your current hospitalist group to gain perspective that will help in your new setting. All programs and hospitals operate differently and have room for quality/process improvement. Interview hospitalists, ask questions, and observe the workflow in your current hospital(s) to help in your new job.

3) Contemplate your career direction: Think strategically about your strengths and plans. Although you will learn an incredible amount about hospital medicine careers after you begin, having a sense of direction will help your new group and its leadership get you where you want to be. Making connections and making your goals known within your new program before you start will put your new career on the right path.

 

 

4) Seek mentors: Having mentors from your prior program and your new program is a key to a healthy and happy career. Choose people you respect and pick their brains about their careers, how they acquired their skills, and how they would advise you to do the same. Good mentors will help you for many years, and the most valuable may be the ones who have known you throughout your residency. Nurture and maintain these relationships even if you are moving on to new horizons. Inquire whether your new program has a mentorship structure or if your new group leader can recommend someone who shares common interests and goals.

5) Study SHM’s Core Com-petencies: Although you may have trepidation about your medical skills and knowledge as you move into uncharted waters, step back and relax. Know that you are prepared. That said, you can always learn more. One excellent resource is The Core Competencies in Hospital Medicine: A Framework for Curriculum Development (available on SHM’s Web site, www.hospitalmedicine.org). This is a set of standards with which programs can teach hospital medicine and you can learn the scope of expectations and competencies for someone in your position.

6) Understand the nuts and bolts of your new program: Although there are many things you will learn on the job, gain an appreciation for some of the following before your first day:

  • Billing: If this is your responsibility, you need to learn a little about this before you start, preferably from one of your future colleagues.
  • Reimbursement structure: Find out how your productivity is tracked and rewarded. You’d be amazed how variable this can be.
  • Time allotment: How are administrative, research, committee and teaching time balanced against your clinical time?

7) Get to know your new hospital: Before hitting the wards it pays to do a little homework on your new workplace. Do you have access to a medical library, journals, UpToDate, or other online databases? If not, do you need to purchase this access on your own? Many programs have academic funds allotted so you can use those resources. Also, familiarize yourself with the local antibiogram, formularies, guidelines, and order sets. Most facilities have tools specific to their hospital. Know how these affect you in your new role. Prior to starting, you will also want to be sufficiently oriented to any computer systems and understand how they’re used for documentation and order entry, and for viewing lab, radiology, and microbiology results.

8) Shadow a hospitalist: Spending a few hours with someone during a typical hospitalist work day will give you an idea of the pace of the work, the layout of the hospital and floors, the medical and ancillary staff you will work with, and the patient population. This will prompt questions you hadn’t thought of previously.

9) Prepare for each specific role: Hospitalists wear many hats, including teaching attending, non-teaching attending, consultant, researcher, committee member, and hospital medicine leader. Each role carries specific responsibilities and expectations. Prior to each new role, train with someone who leads that service or knows the job intimately.

10) Comprehend your benefits: Does your employer have a retirement program? Do they match retirement contributions? How does the malpractice insurance work? A meeting with human resources will usually help you arrange your health, dental, malpractice, and disability insurance prior to your start date. TH

Dr. Chacko is chair of SHM’s young physician committee and the hospitalist program medical director for Preferred Health Partners in New York City. Dr. Markoff is an assistant professor of medicine and associate director of the hospitalist service at the Mount Sinai Hospital in New York City. Dr. Sliwka is a hospitalist and assistant professor of clinical medicine at the University of California, San Francisco Medical Center.

Issue
The Hospitalist - 2008(01)
Issue
The Hospitalist - 2008(01)
Publications
Publications
Article Type
Display Headline
Your First Job
Display Headline
Your First Job
Sections
Disallow All Ads
Content Gating
No Gating (article Unlocked/Free)

In the Literature

Article Type
Changed
Fri, 09/14/2018 - 12:39
Display Headline
In the Literature

Semi-Recumbent Position to Prevent Ventilator-Associated Pneumonia: Is It Possible?

By Joseph Ming Wah Li, MD

Van Nieuwenhoven CA, Vandenbroucke-Grauls C, van Tiel FH, et al. Feasibility and effects of the semirecumbent position to prevent ventilator-associated pneumonia: a randomized study. Crit Care Med. 2006 Feb;34(2):396-402.

Ventilator-associated pneumonia (VAP) is a cause of significant morbidity and mortality among mechanically ventilated patients. Studies with radioactive-labeled enteral feeds have demonstrated an increased frequency of endotracheal aspiration of gastric contents in supine patients. The CDC guidelines for prevention of nosocomial pneumonia advise placement of mechanically ventilated patients in a semi-recumbent position as a VAP prevention measure.

Only one previous study, by Drakulovic and colleagues, has assessed this strategy to prevent VAP.1 That study demonstrated a 75% decrease in the incidence of VAP. But van Nieuwenhoven and colleagues raised two important questions about the findings from the previous study: First, the Drakulovic study placed control patients in a horizontal (zero degrees) position, which is not the standard of care in most ICUs. Most patients are placed at 10 degrees, and this position is elevated as patients are weaned. Second, the Drakulovic study measured patients only once daily but did not monitor their body positions in between the daily measurements.

Nurses always respected the patient’s request for positioning, but a dedicated research nurse restored backrest position to the randomized position whenever possible.

Dr. van Nieuwenhoven and colleagues set out to determine whether it is feasible to keep mechanically ventilated patients in a semi-recumbent position on a continual basis and whether this measure would prevent VAP. This was a prospective multi-centered trial in which mechanically ventilated patients were randomly assigned to the semi-recumbent position with a target backrest elevation of 45 degrees or standard of care (supine position) with a backrest elevation of 10 degrees. They used a transducer with a pendulum, which was placed on the bed frame to measure the backrest elevation every 60 seconds for up to seven days. They calculated a mean degree of elevation for each patient daily. Nurses always respected the patient’s request for positioning, but a dedicated research nurse restored backrest position to the randomized position whenever possible.

Baseline characteristics for both groups were similar. For the supine (control) group, average elevations were 9.8 degrees on day one and 16.1 degrees on day seven. For the semi-recumbent group, average elevations were 28.1 degrees on day one and 22.6 degrees on day seven. There were no significant differences in numbers of patients who developed VAP in either group.

This study suggests that, despite the use of dedicated research nurses to maintain positioning, it may not be possible to keep patients’ backrests elevated to 45 degrees. Keeping patients’ backrests at an elevation of nearly 30 degrees does not appear to prevent VAP more than keeping patients’ backrests at 10 degrees, the present standard of care.

Reference

  1. Drakulovic MB, Torres A, Bauer TT, et al. Supine body position as a risk factor for nosocomial pneumonia in mechanically ventilated patients: a randomised trial. Lancet. 1999;354(9193):1851-1858.

Bar Codes in Medicine: An Opportunity for Quality Improvement

By Alex Carbo, MD

Poon EG, Cina JL, Churchill W, et al. Medication dispensing errors and potential adverse drug events before and after implementing bar code technology in the pharmacy. Ann Intern Med. 2006;145:426-434.

Medication errors and adverse drug events (ADEs) have received much attention in the literature; the use of health information technology to mitigate these errors and ADEs has now been proposed in many areas of healthcare. In an effort to decrease medication-dispensing errors, the U.S. Food and Drug Administration (FDA) mandated bar code use for all medications in hospitals, beginning in April 2006. While this technology has been extensively studied in other industries, there is little data describing its effects in the healthcare system.

 

 

Poon and colleagues set out to evaluate whether implementation of bar code technology reduced dispensing errors and the ADEs that might be caused by these miscalculations. In a before-and-after evaluation, they studied more than 350,000 dispensed medication doses in an academic medical center between February 2003 and September 2004.

In light of the FDA’s mandate regarding bar codes, it seems that every hospital has the opportunity to improve patient safety and decrease medication error rates with the use of bar code technology.

During the bar code conversion process, the hospital pharmacy built a dedicated repackaging center, which was responsible for affixing a bar code to every dose of medication. These medications were then dispensed in three different configurations: two configurations required staff to verify all doses at least once using bar code scanning, and the third configuration—for commonly dispensed medications that could not be accommodated in a standard carousel machine because of their size or need for refrigeration—required scanning only one dose from each batch.

The authors found a 93% to 96% relative reduction in the incidence of target dispensing errors (P<0.001) and an 86% to 97% relative reduction in the incidence of potential ADEs (P<0.001) in the two configurations that required staff to verify all doses by scanning. The greatest reductions were seen in wrong medication errors (56%), wrong strength/dose errors (71%), wrong formulation errors (90%), and expired medication errors (100%).

In the configuration that did not require scanning of every dose, however, there was a 60% relative reduction in the incidence of target dispensing errors (P<0.001), but a 2.4-fold increase in the incidence of target potential ADEs. This included new errors attributable to wrong strength and wrong medication dispensing.

In light of the FDA’s mandate regarding bar codes, it seems that every hospital has the opportunity to improve patient safety and decrease medication error rates with the use of bar code technology. This study suggests that in order to achieve this benefit these systems should be designed to ensure that every medication dose is verified by scanning during the dispensing process.

Evaluation of a Guideline to Guide Resuscitation

By Cindy Lien, MD

Morrison LJ, Visentin LM, Kiss A, et al. Validation of a rule for termination of resuscitation in out-of-hospital cardiac arrest. N Engl J Med. 2006 Aug 3;355(5):478-487.

The survival rate of patients with out-of-hospital cardiac arrest is very low. Thus, guidelines have been developed for termination of resuscitation for those patients who have had no response to advanced cardiac life support provided by emergency medical service (EMS) personnel. Similar guidelines have not yet been developed, however, for situations in which patients receive basic life support from emergency workers trained in the use of an automated external cardiac defibrillator. Patients with little potential for survival are routinely transported to emergency departments, at significant cost to the healthcare system.

Morrison and colleagues present results from the Termination of Resuscitation (TOR) study, a prospective evaluation of a clinical prediction rule for the termination of basic life support by emergency medical personnel trained in the use of automated external defibrillators. The clinical prediction rule, previously developed in a retrospective review of case records from a large urban EMS system, recommends termination of resuscitation if there is no return of spontaneous circulation, no shock administered, and no witness of the arrest by EMS personnel.

When the variable “not witnessed by bystander” was added to the clinical prediction rule, both the positive predictive value and specificity increased to 100%.

In the current study, the authors obtained follow-up data for 1,240 adult patients in Ontario, Canada, who had suffered an arrest of presumed cardiac cause and were subsequently transported to the emergency department after resuscitative efforts. Twenty-four EMS systems participated in the study. The study found that only 0.5% of the patients for whom the clinical prediction rule recommended termination survived (four out of 776 patients). Of the 1,240 total study patients, 41 (3%) survived. The clinical prediction rule recommended continuation of resuscitative efforts for 37 of these 41 patients, resulting in a specificity of 90.2%. The positive predictive value for death was calculated to be 99.5% when termination was recommended.

 

 

The TOR trial also determined whether the addition of other criteria to the original prediction rule could further refine the specificity and positive predictive value. They found that the addition to the criteria of a response time greater than eight minutes increased the positive predictive value and specificity to 99.7% and 97.6%, respectively. When the variable “not witnessed by bystander” was added to the clinical prediction rule, both the positive predictive value and specificity increased to 100%. In other words, no patients survived if they had had a completely unwitnessed arrest, no return of spontaneous circulation, and no shocks delivered.

This study identifies a subpopulation of patients with presumed cardiac arrest for whom termination of resuscitative efforts in the field appears reasonable. The authors note that a survival rate of 1% or less has been suggested in past literature as reflective of medical futility. The TOR investigators acknowledge that their study took place before the 2005 AHA Guidelines for Cardiopulmonary Resuscitation and Emergency Cardiovascular Care were released and that their study protocols were consistent with the 2000 resuscitation guidelines. In light of this information, continued validity testing of the clinical prediction rule under the 2005 AHA protocols is needed.

Nonetheless, it is quite impressive that use of their clinical prediction rule would have resulted in transportation of only 37% of patients (464 of 1,240), rather than 100% of patients, as is currently the practice. If the guidelines described in this article are to be implemented, further studies are necessary to address the training of EMS personnel, who would carry responsibility for terminating resuscitation and notifying families of patients’ deaths.

Prevention of Ventilator-Associated Pneumonia

By Diane Sliwka, MD

Koeman M, van der Ven AJ, Hak E, et al. Oral decontamination with chlorhexidine reduces the incidence of ventilator-associated pneumonia. Am J Respir Crit Care Med. 2006 Jun;173(12):1348-1355. Epub 2006 Apr 7.

Ventilator-associated pneumonia (VAP) is an important nosocomial source of morbidity and mortality. The use of prophylactic antimicrobials to decrease VAP raises concern for antimicrobial resistance. This study evaluates the topical antiseptic chlorhexidine (CHX) as an alternative prophylactic intervention for VAP. CHX has previously been shown to decrease VAP in cardiac surgical patients, but has not been studied in higher risk, long-term-ventilated patients. Because CHX works better for gram-positive organisms, the combination of colistin and CHX (COL + CHX) was also studied for improved gram-negative coverage.

This multi-center, randomized, double-blind, placebo-controlled trial enrolled 385 adult patients. Patients who were expected to be intubated for longer than 48 hours were randomized to 3 arms: CHX alone, CHX + COL, and placebo. Exclusion criteria included known preadmission immunocompromised state, pregnancy, and physical limitation to oral application. Pneumonia was defined by clinical decision-making, which was later confirmed by three blinded intensivists’ reviews of the case records and supported by daily clinical pulmonary infection scores.

The primary endpoint of VAP was diagnosed in 52 of 385 patients: 18% placebo, 13% CHX, and 10% CHX + COL. Rate of VAP in the two treatment groups was lower than placebo and reached statistical significance when compared to placebo. The daily hazard ratio for CHX versus placebo was .352 (95% CI .160, .791); for CHX + COL versus placebo, it was .454 (95% CI .224, .925), showing a 65% and 55% reduction in the rate of pneumonia development. Multivariate analysis of variables such as gender, pulmonary admission diagnosis, colonization at time of admission, and antimicrobial use on admission did not affect the data.

The use of prophylactic antimicrobials to decrease VAP raises concern for antimicrobial resistance.

The secondary endpoint of endotracheal colonization was evaluated by a twice-weekly endotracheal culture. There was no statistically significant difference in colonization among the three groups in the first (days 1-4) or third (days 9-12) time frames. During the second time frame (days 5-8), there was a statistically significant decrease in colonization for the CHX + COL treatment group when compared to both placebo (16% versus 40% p<.007) and to CHX (16% versus 38%, p<.011); this decrease is thought to be due to gram-negative coverage by COL.

 

 

The secondary endpoint of oropharyngeal colonization was evaluated for 87% of all patient days. CHX and CHX + COL were similarly effective for gram-positive bacteria when compared to placebo, with 30% and 27% reduction in rates of colonization, respectively: HR 0.695 for CHX (95% CI, 0.606, 0.796; p < 0.001) and 0.732 (95% CI, 0.640, 0.838; p < 0.001) for CHX + COL. The CHX + COL combination was more effective for gram-negative bacteria: daily HR .534 (95% CI, 0.455, 0.626; p <0.001) alone with a 47% reduction in gram-negative colonization compared to CHX.

No difference was seen in ICU mortality, duration of mechanical ventilation, or duration of ICU stay. One adverse event (tongue swelling) occurred in the CHX + COL group.

Limitations of the study include the following:

  • Daily assessments on all patients were not performed;
  • The placebo group had more males and more infections on admission than the other two groups, raising the question of randomization error;
  • Clinical versus quantitative diagnosis of pneumonia may overestimate VAP in this study;
  • It is not known how many patients were not enrolled in the study due to short anticipated ventilator times, but who later had prolonged ventilations; and
  • The lack of effect on ventilator time, ICU length of stay, and mortality raises the question of the significance of these findings.

Despite these limitations, the low cost of these treatments, minimal adverse events, low risk of promoting significant antimicrobial resistance, and the finding of decreased VAP and bacterial colonization risk shown in this study support the potential benefit of topical decontamination with CHX and COL in conjunction with other measures of VAP prevention. TH

Reference

  1. De Riso AJ II, Ladowski JS, Dillon TA, et al. Chlorhexidine gluconate 0.12% oral rinse reduces the incidence of total nosocomial respiratory infection and nonprophylactic systemic antibiotic use in patients undergoing heart surgery. Chest. 1996;109:1556-1561.
Issue
The Hospitalist - 2006(12)
Publications
Sections

Semi-Recumbent Position to Prevent Ventilator-Associated Pneumonia: Is It Possible?

By Joseph Ming Wah Li, MD

Van Nieuwenhoven CA, Vandenbroucke-Grauls C, van Tiel FH, et al. Feasibility and effects of the semirecumbent position to prevent ventilator-associated pneumonia: a randomized study. Crit Care Med. 2006 Feb;34(2):396-402.

Ventilator-associated pneumonia (VAP) is a cause of significant morbidity and mortality among mechanically ventilated patients. Studies with radioactive-labeled enteral feeds have demonstrated an increased frequency of endotracheal aspiration of gastric contents in supine patients. The CDC guidelines for prevention of nosocomial pneumonia advise placement of mechanically ventilated patients in a semi-recumbent position as a VAP prevention measure.

Only one previous study, by Drakulovic and colleagues, has assessed this strategy to prevent VAP.1 That study demonstrated a 75% decrease in the incidence of VAP. But van Nieuwenhoven and colleagues raised two important questions about the findings from the previous study: First, the Drakulovic study placed control patients in a horizontal (zero degrees) position, which is not the standard of care in most ICUs. Most patients are placed at 10 degrees, and this position is elevated as patients are weaned. Second, the Drakulovic study measured patients only once daily but did not monitor their body positions in between the daily measurements.

Nurses always respected the patient’s request for positioning, but a dedicated research nurse restored backrest position to the randomized position whenever possible.

Dr. van Nieuwenhoven and colleagues set out to determine whether it is feasible to keep mechanically ventilated patients in a semi-recumbent position on a continual basis and whether this measure would prevent VAP. This was a prospective multi-centered trial in which mechanically ventilated patients were randomly assigned to the semi-recumbent position with a target backrest elevation of 45 degrees or standard of care (supine position) with a backrest elevation of 10 degrees. They used a transducer with a pendulum, which was placed on the bed frame to measure the backrest elevation every 60 seconds for up to seven days. They calculated a mean degree of elevation for each patient daily. Nurses always respected the patient’s request for positioning, but a dedicated research nurse restored backrest position to the randomized position whenever possible.

Baseline characteristics for both groups were similar. For the supine (control) group, average elevations were 9.8 degrees on day one and 16.1 degrees on day seven. For the semi-recumbent group, average elevations were 28.1 degrees on day one and 22.6 degrees on day seven. There were no significant differences in numbers of patients who developed VAP in either group.

This study suggests that, despite the use of dedicated research nurses to maintain positioning, it may not be possible to keep patients’ backrests elevated to 45 degrees. Keeping patients’ backrests at an elevation of nearly 30 degrees does not appear to prevent VAP more than keeping patients’ backrests at 10 degrees, the present standard of care.

Reference

  1. Drakulovic MB, Torres A, Bauer TT, et al. Supine body position as a risk factor for nosocomial pneumonia in mechanically ventilated patients: a randomised trial. Lancet. 1999;354(9193):1851-1858.

Bar Codes in Medicine: An Opportunity for Quality Improvement

By Alex Carbo, MD

Poon EG, Cina JL, Churchill W, et al. Medication dispensing errors and potential adverse drug events before and after implementing bar code technology in the pharmacy. Ann Intern Med. 2006;145:426-434.

Medication errors and adverse drug events (ADEs) have received much attention in the literature; the use of health information technology to mitigate these errors and ADEs has now been proposed in many areas of healthcare. In an effort to decrease medication-dispensing errors, the U.S. Food and Drug Administration (FDA) mandated bar code use for all medications in hospitals, beginning in April 2006. While this technology has been extensively studied in other industries, there is little data describing its effects in the healthcare system.

 

 

Poon and colleagues set out to evaluate whether implementation of bar code technology reduced dispensing errors and the ADEs that might be caused by these miscalculations. In a before-and-after evaluation, they studied more than 350,000 dispensed medication doses in an academic medical center between February 2003 and September 2004.

In light of the FDA’s mandate regarding bar codes, it seems that every hospital has the opportunity to improve patient safety and decrease medication error rates with the use of bar code technology.

During the bar code conversion process, the hospital pharmacy built a dedicated repackaging center, which was responsible for affixing a bar code to every dose of medication. These medications were then dispensed in three different configurations: two configurations required staff to verify all doses at least once using bar code scanning, and the third configuration—for commonly dispensed medications that could not be accommodated in a standard carousel machine because of their size or need for refrigeration—required scanning only one dose from each batch.

The authors found a 93% to 96% relative reduction in the incidence of target dispensing errors (P<0.001) and an 86% to 97% relative reduction in the incidence of potential ADEs (P<0.001) in the two configurations that required staff to verify all doses by scanning. The greatest reductions were seen in wrong medication errors (56%), wrong strength/dose errors (71%), wrong formulation errors (90%), and expired medication errors (100%).

In the configuration that did not require scanning of every dose, however, there was a 60% relative reduction in the incidence of target dispensing errors (P<0.001), but a 2.4-fold increase in the incidence of target potential ADEs. This included new errors attributable to wrong strength and wrong medication dispensing.

In light of the FDA’s mandate regarding bar codes, it seems that every hospital has the opportunity to improve patient safety and decrease medication error rates with the use of bar code technology. This study suggests that in order to achieve this benefit these systems should be designed to ensure that every medication dose is verified by scanning during the dispensing process.

Evaluation of a Guideline to Guide Resuscitation

By Cindy Lien, MD

Morrison LJ, Visentin LM, Kiss A, et al. Validation of a rule for termination of resuscitation in out-of-hospital cardiac arrest. N Engl J Med. 2006 Aug 3;355(5):478-487.

The survival rate of patients with out-of-hospital cardiac arrest is very low. Thus, guidelines have been developed for termination of resuscitation for those patients who have had no response to advanced cardiac life support provided by emergency medical service (EMS) personnel. Similar guidelines have not yet been developed, however, for situations in which patients receive basic life support from emergency workers trained in the use of an automated external cardiac defibrillator. Patients with little potential for survival are routinely transported to emergency departments, at significant cost to the healthcare system.

Morrison and colleagues present results from the Termination of Resuscitation (TOR) study, a prospective evaluation of a clinical prediction rule for the termination of basic life support by emergency medical personnel trained in the use of automated external defibrillators. The clinical prediction rule, previously developed in a retrospective review of case records from a large urban EMS system, recommends termination of resuscitation if there is no return of spontaneous circulation, no shock administered, and no witness of the arrest by EMS personnel.

When the variable “not witnessed by bystander” was added to the clinical prediction rule, both the positive predictive value and specificity increased to 100%.

In the current study, the authors obtained follow-up data for 1,240 adult patients in Ontario, Canada, who had suffered an arrest of presumed cardiac cause and were subsequently transported to the emergency department after resuscitative efforts. Twenty-four EMS systems participated in the study. The study found that only 0.5% of the patients for whom the clinical prediction rule recommended termination survived (four out of 776 patients). Of the 1,240 total study patients, 41 (3%) survived. The clinical prediction rule recommended continuation of resuscitative efforts for 37 of these 41 patients, resulting in a specificity of 90.2%. The positive predictive value for death was calculated to be 99.5% when termination was recommended.

 

 

The TOR trial also determined whether the addition of other criteria to the original prediction rule could further refine the specificity and positive predictive value. They found that the addition to the criteria of a response time greater than eight minutes increased the positive predictive value and specificity to 99.7% and 97.6%, respectively. When the variable “not witnessed by bystander” was added to the clinical prediction rule, both the positive predictive value and specificity increased to 100%. In other words, no patients survived if they had had a completely unwitnessed arrest, no return of spontaneous circulation, and no shocks delivered.

This study identifies a subpopulation of patients with presumed cardiac arrest for whom termination of resuscitative efforts in the field appears reasonable. The authors note that a survival rate of 1% or less has been suggested in past literature as reflective of medical futility. The TOR investigators acknowledge that their study took place before the 2005 AHA Guidelines for Cardiopulmonary Resuscitation and Emergency Cardiovascular Care were released and that their study protocols were consistent with the 2000 resuscitation guidelines. In light of this information, continued validity testing of the clinical prediction rule under the 2005 AHA protocols is needed.

Nonetheless, it is quite impressive that use of their clinical prediction rule would have resulted in transportation of only 37% of patients (464 of 1,240), rather than 100% of patients, as is currently the practice. If the guidelines described in this article are to be implemented, further studies are necessary to address the training of EMS personnel, who would carry responsibility for terminating resuscitation and notifying families of patients’ deaths.

Prevention of Ventilator-Associated Pneumonia

By Diane Sliwka, MD

Koeman M, van der Ven AJ, Hak E, et al. Oral decontamination with chlorhexidine reduces the incidence of ventilator-associated pneumonia. Am J Respir Crit Care Med. 2006 Jun;173(12):1348-1355. Epub 2006 Apr 7.

Ventilator-associated pneumonia (VAP) is an important nosocomial source of morbidity and mortality. The use of prophylactic antimicrobials to decrease VAP raises concern for antimicrobial resistance. This study evaluates the topical antiseptic chlorhexidine (CHX) as an alternative prophylactic intervention for VAP. CHX has previously been shown to decrease VAP in cardiac surgical patients, but has not been studied in higher risk, long-term-ventilated patients. Because CHX works better for gram-positive organisms, the combination of colistin and CHX (COL + CHX) was also studied for improved gram-negative coverage.

This multi-center, randomized, double-blind, placebo-controlled trial enrolled 385 adult patients. Patients who were expected to be intubated for longer than 48 hours were randomized to 3 arms: CHX alone, CHX + COL, and placebo. Exclusion criteria included known preadmission immunocompromised state, pregnancy, and physical limitation to oral application. Pneumonia was defined by clinical decision-making, which was later confirmed by three blinded intensivists’ reviews of the case records and supported by daily clinical pulmonary infection scores.

The primary endpoint of VAP was diagnosed in 52 of 385 patients: 18% placebo, 13% CHX, and 10% CHX + COL. Rate of VAP in the two treatment groups was lower than placebo and reached statistical significance when compared to placebo. The daily hazard ratio for CHX versus placebo was .352 (95% CI .160, .791); for CHX + COL versus placebo, it was .454 (95% CI .224, .925), showing a 65% and 55% reduction in the rate of pneumonia development. Multivariate analysis of variables such as gender, pulmonary admission diagnosis, colonization at time of admission, and antimicrobial use on admission did not affect the data.

The use of prophylactic antimicrobials to decrease VAP raises concern for antimicrobial resistance.

The secondary endpoint of endotracheal colonization was evaluated by a twice-weekly endotracheal culture. There was no statistically significant difference in colonization among the three groups in the first (days 1-4) or third (days 9-12) time frames. During the second time frame (days 5-8), there was a statistically significant decrease in colonization for the CHX + COL treatment group when compared to both placebo (16% versus 40% p<.007) and to CHX (16% versus 38%, p<.011); this decrease is thought to be due to gram-negative coverage by COL.

 

 

The secondary endpoint of oropharyngeal colonization was evaluated for 87% of all patient days. CHX and CHX + COL were similarly effective for gram-positive bacteria when compared to placebo, with 30% and 27% reduction in rates of colonization, respectively: HR 0.695 for CHX (95% CI, 0.606, 0.796; p < 0.001) and 0.732 (95% CI, 0.640, 0.838; p < 0.001) for CHX + COL. The CHX + COL combination was more effective for gram-negative bacteria: daily HR .534 (95% CI, 0.455, 0.626; p <0.001) alone with a 47% reduction in gram-negative colonization compared to CHX.

No difference was seen in ICU mortality, duration of mechanical ventilation, or duration of ICU stay. One adverse event (tongue swelling) occurred in the CHX + COL group.

Limitations of the study include the following:

  • Daily assessments on all patients were not performed;
  • The placebo group had more males and more infections on admission than the other two groups, raising the question of randomization error;
  • Clinical versus quantitative diagnosis of pneumonia may overestimate VAP in this study;
  • It is not known how many patients were not enrolled in the study due to short anticipated ventilator times, but who later had prolonged ventilations; and
  • The lack of effect on ventilator time, ICU length of stay, and mortality raises the question of the significance of these findings.

Despite these limitations, the low cost of these treatments, minimal adverse events, low risk of promoting significant antimicrobial resistance, and the finding of decreased VAP and bacterial colonization risk shown in this study support the potential benefit of topical decontamination with CHX and COL in conjunction with other measures of VAP prevention. TH

Reference

  1. De Riso AJ II, Ladowski JS, Dillon TA, et al. Chlorhexidine gluconate 0.12% oral rinse reduces the incidence of total nosocomial respiratory infection and nonprophylactic systemic antibiotic use in patients undergoing heart surgery. Chest. 1996;109:1556-1561.

Semi-Recumbent Position to Prevent Ventilator-Associated Pneumonia: Is It Possible?

By Joseph Ming Wah Li, MD

Van Nieuwenhoven CA, Vandenbroucke-Grauls C, van Tiel FH, et al. Feasibility and effects of the semirecumbent position to prevent ventilator-associated pneumonia: a randomized study. Crit Care Med. 2006 Feb;34(2):396-402.

Ventilator-associated pneumonia (VAP) is a cause of significant morbidity and mortality among mechanically ventilated patients. Studies with radioactive-labeled enteral feeds have demonstrated an increased frequency of endotracheal aspiration of gastric contents in supine patients. The CDC guidelines for prevention of nosocomial pneumonia advise placement of mechanically ventilated patients in a semi-recumbent position as a VAP prevention measure.

Only one previous study, by Drakulovic and colleagues, has assessed this strategy to prevent VAP.1 That study demonstrated a 75% decrease in the incidence of VAP. But van Nieuwenhoven and colleagues raised two important questions about the findings from the previous study: First, the Drakulovic study placed control patients in a horizontal (zero degrees) position, which is not the standard of care in most ICUs. Most patients are placed at 10 degrees, and this position is elevated as patients are weaned. Second, the Drakulovic study measured patients only once daily but did not monitor their body positions in between the daily measurements.

Nurses always respected the patient’s request for positioning, but a dedicated research nurse restored backrest position to the randomized position whenever possible.

Dr. van Nieuwenhoven and colleagues set out to determine whether it is feasible to keep mechanically ventilated patients in a semi-recumbent position on a continual basis and whether this measure would prevent VAP. This was a prospective multi-centered trial in which mechanically ventilated patients were randomly assigned to the semi-recumbent position with a target backrest elevation of 45 degrees or standard of care (supine position) with a backrest elevation of 10 degrees. They used a transducer with a pendulum, which was placed on the bed frame to measure the backrest elevation every 60 seconds for up to seven days. They calculated a mean degree of elevation for each patient daily. Nurses always respected the patient’s request for positioning, but a dedicated research nurse restored backrest position to the randomized position whenever possible.

Baseline characteristics for both groups were similar. For the supine (control) group, average elevations were 9.8 degrees on day one and 16.1 degrees on day seven. For the semi-recumbent group, average elevations were 28.1 degrees on day one and 22.6 degrees on day seven. There were no significant differences in numbers of patients who developed VAP in either group.

This study suggests that, despite the use of dedicated research nurses to maintain positioning, it may not be possible to keep patients’ backrests elevated to 45 degrees. Keeping patients’ backrests at an elevation of nearly 30 degrees does not appear to prevent VAP more than keeping patients’ backrests at 10 degrees, the present standard of care.

Reference

  1. Drakulovic MB, Torres A, Bauer TT, et al. Supine body position as a risk factor for nosocomial pneumonia in mechanically ventilated patients: a randomised trial. Lancet. 1999;354(9193):1851-1858.

Bar Codes in Medicine: An Opportunity for Quality Improvement

By Alex Carbo, MD

Poon EG, Cina JL, Churchill W, et al. Medication dispensing errors and potential adverse drug events before and after implementing bar code technology in the pharmacy. Ann Intern Med. 2006;145:426-434.

Medication errors and adverse drug events (ADEs) have received much attention in the literature; the use of health information technology to mitigate these errors and ADEs has now been proposed in many areas of healthcare. In an effort to decrease medication-dispensing errors, the U.S. Food and Drug Administration (FDA) mandated bar code use for all medications in hospitals, beginning in April 2006. While this technology has been extensively studied in other industries, there is little data describing its effects in the healthcare system.

 

 

Poon and colleagues set out to evaluate whether implementation of bar code technology reduced dispensing errors and the ADEs that might be caused by these miscalculations. In a before-and-after evaluation, they studied more than 350,000 dispensed medication doses in an academic medical center between February 2003 and September 2004.

In light of the FDA’s mandate regarding bar codes, it seems that every hospital has the opportunity to improve patient safety and decrease medication error rates with the use of bar code technology.

During the bar code conversion process, the hospital pharmacy built a dedicated repackaging center, which was responsible for affixing a bar code to every dose of medication. These medications were then dispensed in three different configurations: two configurations required staff to verify all doses at least once using bar code scanning, and the third configuration—for commonly dispensed medications that could not be accommodated in a standard carousel machine because of their size or need for refrigeration—required scanning only one dose from each batch.

The authors found a 93% to 96% relative reduction in the incidence of target dispensing errors (P<0.001) and an 86% to 97% relative reduction in the incidence of potential ADEs (P<0.001) in the two configurations that required staff to verify all doses by scanning. The greatest reductions were seen in wrong medication errors (56%), wrong strength/dose errors (71%), wrong formulation errors (90%), and expired medication errors (100%).

In the configuration that did not require scanning of every dose, however, there was a 60% relative reduction in the incidence of target dispensing errors (P<0.001), but a 2.4-fold increase in the incidence of target potential ADEs. This included new errors attributable to wrong strength and wrong medication dispensing.

In light of the FDA’s mandate regarding bar codes, it seems that every hospital has the opportunity to improve patient safety and decrease medication error rates with the use of bar code technology. This study suggests that in order to achieve this benefit these systems should be designed to ensure that every medication dose is verified by scanning during the dispensing process.

Evaluation of a Guideline to Guide Resuscitation

By Cindy Lien, MD

Morrison LJ, Visentin LM, Kiss A, et al. Validation of a rule for termination of resuscitation in out-of-hospital cardiac arrest. N Engl J Med. 2006 Aug 3;355(5):478-487.

The survival rate of patients with out-of-hospital cardiac arrest is very low. Thus, guidelines have been developed for termination of resuscitation for those patients who have had no response to advanced cardiac life support provided by emergency medical service (EMS) personnel. Similar guidelines have not yet been developed, however, for situations in which patients receive basic life support from emergency workers trained in the use of an automated external cardiac defibrillator. Patients with little potential for survival are routinely transported to emergency departments, at significant cost to the healthcare system.

Morrison and colleagues present results from the Termination of Resuscitation (TOR) study, a prospective evaluation of a clinical prediction rule for the termination of basic life support by emergency medical personnel trained in the use of automated external defibrillators. The clinical prediction rule, previously developed in a retrospective review of case records from a large urban EMS system, recommends termination of resuscitation if there is no return of spontaneous circulation, no shock administered, and no witness of the arrest by EMS personnel.

When the variable “not witnessed by bystander” was added to the clinical prediction rule, both the positive predictive value and specificity increased to 100%.

In the current study, the authors obtained follow-up data for 1,240 adult patients in Ontario, Canada, who had suffered an arrest of presumed cardiac cause and were subsequently transported to the emergency department after resuscitative efforts. Twenty-four EMS systems participated in the study. The study found that only 0.5% of the patients for whom the clinical prediction rule recommended termination survived (four out of 776 patients). Of the 1,240 total study patients, 41 (3%) survived. The clinical prediction rule recommended continuation of resuscitative efforts for 37 of these 41 patients, resulting in a specificity of 90.2%. The positive predictive value for death was calculated to be 99.5% when termination was recommended.

 

 

The TOR trial also determined whether the addition of other criteria to the original prediction rule could further refine the specificity and positive predictive value. They found that the addition to the criteria of a response time greater than eight minutes increased the positive predictive value and specificity to 99.7% and 97.6%, respectively. When the variable “not witnessed by bystander” was added to the clinical prediction rule, both the positive predictive value and specificity increased to 100%. In other words, no patients survived if they had had a completely unwitnessed arrest, no return of spontaneous circulation, and no shocks delivered.

This study identifies a subpopulation of patients with presumed cardiac arrest for whom termination of resuscitative efforts in the field appears reasonable. The authors note that a survival rate of 1% or less has been suggested in past literature as reflective of medical futility. The TOR investigators acknowledge that their study took place before the 2005 AHA Guidelines for Cardiopulmonary Resuscitation and Emergency Cardiovascular Care were released and that their study protocols were consistent with the 2000 resuscitation guidelines. In light of this information, continued validity testing of the clinical prediction rule under the 2005 AHA protocols is needed.

Nonetheless, it is quite impressive that use of their clinical prediction rule would have resulted in transportation of only 37% of patients (464 of 1,240), rather than 100% of patients, as is currently the practice. If the guidelines described in this article are to be implemented, further studies are necessary to address the training of EMS personnel, who would carry responsibility for terminating resuscitation and notifying families of patients’ deaths.

Prevention of Ventilator-Associated Pneumonia

By Diane Sliwka, MD

Koeman M, van der Ven AJ, Hak E, et al. Oral decontamination with chlorhexidine reduces the incidence of ventilator-associated pneumonia. Am J Respir Crit Care Med. 2006 Jun;173(12):1348-1355. Epub 2006 Apr 7.

Ventilator-associated pneumonia (VAP) is an important nosocomial source of morbidity and mortality. The use of prophylactic antimicrobials to decrease VAP raises concern for antimicrobial resistance. This study evaluates the topical antiseptic chlorhexidine (CHX) as an alternative prophylactic intervention for VAP. CHX has previously been shown to decrease VAP in cardiac surgical patients, but has not been studied in higher risk, long-term-ventilated patients. Because CHX works better for gram-positive organisms, the combination of colistin and CHX (COL + CHX) was also studied for improved gram-negative coverage.

This multi-center, randomized, double-blind, placebo-controlled trial enrolled 385 adult patients. Patients who were expected to be intubated for longer than 48 hours were randomized to 3 arms: CHX alone, CHX + COL, and placebo. Exclusion criteria included known preadmission immunocompromised state, pregnancy, and physical limitation to oral application. Pneumonia was defined by clinical decision-making, which was later confirmed by three blinded intensivists’ reviews of the case records and supported by daily clinical pulmonary infection scores.

The primary endpoint of VAP was diagnosed in 52 of 385 patients: 18% placebo, 13% CHX, and 10% CHX + COL. Rate of VAP in the two treatment groups was lower than placebo and reached statistical significance when compared to placebo. The daily hazard ratio for CHX versus placebo was .352 (95% CI .160, .791); for CHX + COL versus placebo, it was .454 (95% CI .224, .925), showing a 65% and 55% reduction in the rate of pneumonia development. Multivariate analysis of variables such as gender, pulmonary admission diagnosis, colonization at time of admission, and antimicrobial use on admission did not affect the data.

The use of prophylactic antimicrobials to decrease VAP raises concern for antimicrobial resistance.

The secondary endpoint of endotracheal colonization was evaluated by a twice-weekly endotracheal culture. There was no statistically significant difference in colonization among the three groups in the first (days 1-4) or third (days 9-12) time frames. During the second time frame (days 5-8), there was a statistically significant decrease in colonization for the CHX + COL treatment group when compared to both placebo (16% versus 40% p<.007) and to CHX (16% versus 38%, p<.011); this decrease is thought to be due to gram-negative coverage by COL.

 

 

The secondary endpoint of oropharyngeal colonization was evaluated for 87% of all patient days. CHX and CHX + COL were similarly effective for gram-positive bacteria when compared to placebo, with 30% and 27% reduction in rates of colonization, respectively: HR 0.695 for CHX (95% CI, 0.606, 0.796; p < 0.001) and 0.732 (95% CI, 0.640, 0.838; p < 0.001) for CHX + COL. The CHX + COL combination was more effective for gram-negative bacteria: daily HR .534 (95% CI, 0.455, 0.626; p <0.001) alone with a 47% reduction in gram-negative colonization compared to CHX.

No difference was seen in ICU mortality, duration of mechanical ventilation, or duration of ICU stay. One adverse event (tongue swelling) occurred in the CHX + COL group.

Limitations of the study include the following:

  • Daily assessments on all patients were not performed;
  • The placebo group had more males and more infections on admission than the other two groups, raising the question of randomization error;
  • Clinical versus quantitative diagnosis of pneumonia may overestimate VAP in this study;
  • It is not known how many patients were not enrolled in the study due to short anticipated ventilator times, but who later had prolonged ventilations; and
  • The lack of effect on ventilator time, ICU length of stay, and mortality raises the question of the significance of these findings.

Despite these limitations, the low cost of these treatments, minimal adverse events, low risk of promoting significant antimicrobial resistance, and the finding of decreased VAP and bacterial colonization risk shown in this study support the potential benefit of topical decontamination with CHX and COL in conjunction with other measures of VAP prevention. TH

Reference

  1. De Riso AJ II, Ladowski JS, Dillon TA, et al. Chlorhexidine gluconate 0.12% oral rinse reduces the incidence of total nosocomial respiratory infection and nonprophylactic systemic antibiotic use in patients undergoing heart surgery. Chest. 1996;109:1556-1561.
Issue
The Hospitalist - 2006(12)
Issue
The Hospitalist - 2006(12)
Publications
Publications
Article Type
Display Headline
In the Literature
Display Headline
In the Literature
Sections
Disallow All Ads
Content Gating
No Gating (article Unlocked/Free)